Sie sind auf Seite 1von 87

GASEOUS ST AT E Rg - 2015 - 2017

IIT – JEE SYLLABUS

Absolute scale of temperature, ideal gas equation; Deviation from ideality, van der Waals equation;
Kinetic theory of gases, average, root mean square and most probable velocities and their relation
with temperature; Law of partial pressures; Vapour pressure; Diffusion of gases.

CONTENTS

1. Theory 2 – 28

2. Solved Problems (Objective) 28 – 32

3. Solved Problems (Subjective) 33 – 40

4. Foundation Builders (Objective) 41 – 50

5. Foundation Builders (Subjective) 51 – 54

6. Get Equipped for JEE (MAIN) 55 – 57

7. Get Equipped for JEE (ADVANCED) 58 – 69

8. Window To IIT-JEE (MAIN) 70 – 71

9. Window To IIT-JEE (ADVANCED) 72 – 79

10. Expertise Attainers 80 – 82

11. Answer Keys 83 – 86

Gaseous State : Tentative Lecture Flow


(Board Syllabus & Booklet Discussion Included)
Lecture –1 Ideal gas laws and Ideal gas equation

Lecture – 2 Problems on ideal gas equation, Dalton’s law

Lecture – 3 Graham’s law


Lecture – 4 Eudiometry
Lecture – 5 KTG
Lecture – 6 Real gas equation

CENTERS: MUMBAI / DELHI / AKOLA / KOLKATA / LUCKNOW / NASHIK / GOA #


GASEOUS ST AT E Rg - 2015 - 2017

Lecture – 7 Compressibility factor, virial equation

1. INTRODUCTION
The various kinds of substance that make up matter can be divided roughly into three categories,
namely, gases, liquids and solids. These are called as three states of matter. These states can be considered to
arise as a result of competition between two opposing molecular forces, namely, the force of attraction
which tends to hold the molecules together, and the disruptive force due to the thermal energy of molecules.
If the disruptive forces due to thermal energy are much greater than the forces of attraction, then we have
matter in its gaseous state. Molecules in the gaseous state move with very large speeds and the forces of
attraction amongst them are not sufficient to bind the molecules at one place, with the result that the
molecules move practically independent of one another. Because of this feature, gases are characterized by
marked sensitivity of volume change with change of temperature and pressure. There exists no boundary
surface and, therefore, gases tend to fill completely any available space, i.e. they do not possess a fixed
volume. If the forces of attraction are greater than the disruptive forces due to thermal energy, we have
matter in the liquid state. Molecules in the liquid state too have kinetic energy but they cannot go very far
away because of the larger forces of attraction amongst them. Due to this feature, liquids have definite
volume, but they do not have definite shape. They take the shape of the vessel in which they are placed. In
general, liquids are denser and less compressible than gases. If the forces of attraction between molecules
are much greater than the disruptive forces due to thermal energy, the positions of the molecules remain
fixed and we have the solid state of matter. The molecules in the solid state, therefore, do not possess any
translational energy, but have only vibrational energy since they can vibrate about their mean positions.
Extremely large forces of attraction exist amongst them. That is why solids differ markedly from liquids and
gases in respect of size, shape and volume. Solids, in general, have definite size, shape and volume.

Gaseous state is the simplest state of matter. Throughout our life we remain immersed in the ocean of
air which is a mixture of gases. We spend our life in the lowermost layer of the atmosphere called
troposphere, which is held to the surface of the earth by gravitational force. The thin layer of atmosphere is
vital to our life. It shields us from harmful radiations and contains substances like dioxygen, dinitrogen,
carbon dioxide, water vapour, etc. Let us now focus our attention on the behavior of substances which exist
in the gaseous state under normal conditions of temperature and pressure.

The gaseous state is characterized by the following physical properties.


 Gases are highly compressible.
 Gases exert pressure equally in all directions.
 Gases have much lower density than the solids and liquids.
 The volume and the shape of gases are not fixed. These assume volume and shape of the container.
 Gases mix evenly and completely in all proportions without any mechanical aid.

CENTERS: MUMBAI / DELHI / AKOLA / KOLKATA / LUCKNOW / NASHIK / GOA #


GASEOUS ST AT E Rg - 2015 - 2017

2. EXPERIMENTAL GAS LAWS


2.1 Basic parameters associated with gas
The characteristics of gases are described fully in terms of four parameters (measurable properties)

(i) Pressure:- It is exerted by gas due to collision of randomly moving gas molecules with the walls of the
container. Pressure can be measured in following units:
Pascal: - It is SI unit for pressure; Pascal is very small amount of pressure
o (10mg weight on 1cm2 area)
o 1 Pa = 1 N/m2
Atm: - atm is pressure exerted by mass of the atmosphere on the earth’s surface
Various units of pressure can be interrelated as follows
1 atm = 76 cm of Hg = 760 torr = 101325 Pa = 1.01325 bar

(ii) Volume :- It represents free volume available for motion of gas particles
Various units of volume are interrelated as follows
1 L = 1000 ml = 10-3 m3 = 1000 cc

(iii) Temperature :- Units of temperature are interrelated as follws


T K = T0 C + 273

(iv) Amount of gas :- Generally measured in moles of gas

Simplicity of gases is due to the fact that the forces of interaction between their molecules are
negligible. Their behavior is governed by same general laws, which were discovered as a result of their
experimental studies. These laws are relationships between measurable properties of gases. Some of
these properties like pressure, volume, temperature and mass are very important because relationships
between these variables describe state of the gas. Inter-dependence of these variables leads to the
formulation of gas laws.

2.2 Boyle’s Law


In 1662, Robert Boyle discovered that there existed a relation between the pressure and
the volume of a fixed amount of gas at a fixed temperature. In his experiment, he discovered that the
product of Pressure & Volume of a fixed amount of gas at a fixed temperature was approximately a
constant.
1
V (if T and n constant).  PV = Constant
P

CENTERS: MUMBAI / DELHI / AKOLA / KOLKATA / LUCKNOW / NASHIK / GOA #


GASEOUS ST AT E Rg - 2015 - 2017
Graphical Representation: - Boyle’s law can be graphically represented in following ways

T e m p e ra tu re -c o n s ta n t T e m p e ra tu re -c o n s ta n t
T = c o n s ta n t
M a s s = c o n s ta n t
M a s s -c o n s ta n t M a s s -c o n s ta n t

P V PV

V 1 /P P

T 1 T 1
T 2 T
(T 1 > T 2 > T 3) 2
T 3 lo g P
T
P P
3
(T 1 > T 2 > T 3) T PV
1
T2 (T 1> T 2 > T 3)
T3 lo g V
1 /V
V P

Example 1: A 1.1 L flask containing nitrogen at a pressure of 710 mm is connected to an evacuated


flask of unknown volume. The nitrogen, which acts ideally is allowed to expand into the
combined system of both the flasks isothermally. If the final pressure of nitrogen is
583 mm, determine the volume of evacuated flask.
Solution: Applying Boyle’s law: p1V1 = p2V2
Let V be the volume of evacuated flask
 710  1.1 = 583 (1.1 + V)
V = 0.24 L

2.3 Charles law

Charles made measurements of the volume of a fixed mass of a gas at various temperatures under the
condition of constant pressure and found that the volume of the gas is a linear function of the temperature.
Experiments have shown that when 273 ml sample of a gas at 0oC is heated to 1oC, its volume
increases by 1 ml, i.e., it becomes 274 ml. At 10oC, the volume increases to 283 ml if the pressure remains
constant in both cases. Similarly, when 273 ml sample of gas at 0oC is cooled to −1oC, its volume decreases
to 272 ml while at −10oC the volume decreases to 263 ml if the pressure remains constant.

CENTERS: MUMBAI / DELHI / AKOLA / KOLKATA / LUCKNOW / NASHIK / GOA #


GASEOUS ST AT E Rg - 2015 - 2017
Thus, all gases expand or contract by the same fraction of their volumes at 0oC per degree change of
temperature, i.e., for each degree change of temperature, the volume of a sample of a gas changes by the
1
fraction of its volume at 0oC.
273
Let the volume of a given amount of a gas be Vo at 0oC. The temperature is increased by toC and the new
volume becomes Vt.
V0 � t �
Thus, Vt = V0 + �t = V0 �
1+ �
273 � 273 �
�273 + t �
or Vt = V0 � � (since K = oC + 273) … Eq(i)
� 273 �
by substituting T for 273 + t and To for 273 in Eq. (i),
V �T Vt V0 V
Vt = 0 or = or = constant (if pressure is kept constant)
T0 T T0 T
This is Charles law. It can be stated as follows:
The volume of a given amount of a gas at constant pressure varies directly as its absolute
temperature.
V  T (if pressure is kept constant)
P1
P2 > P1
P2
V

Since volume is directly proportional to Kelvin temperature, the volume of a gas should theoretically
be zero at Kelvin zero. However, gases liquefy and then solidify before this low temperature is reached. In
fact, no substance exists as a gas at a temperature near Kelvin zero, through the straight-line plots can be
extrapolated to zero volume. The temperature that corresponds to zero volume is −273.15oC.

 Can you guess how the graph of volume vs Temperature (0C) will look like

Example2: A flask is of a capacity of 1 liter. What volume of air will escape from the flask if it is
heated from 270C to 370C? Assume pressure is constant.
Solution Since pressure is constant
1/300=V1/310
V1 = 1.0333 litres
Since capacity is 1 liter
Thus Volume escaped = 1.0333-1 = 0.0333 L

CENTERS: MUMBAI / DELHI / AKOLA / KOLKATA / LUCKNOW / NASHIK / GOA #


GASEOUS ST AT E Rg - 2015 - 2017

2.4 Gay Lussac’s Law


It relates the pressure and absolute temperature of a given mass of a gas at constant volume. Volume
1
remaining constant, the pressure of a given mass of a gas increases or decreases by of its pressure
273
at 0oC per degree change of temperature.
P0 �t � t �
Pt = P0 + or Pt = P0 �
1+ �
273 � 273 �
�273 + t � T
or Pt = P0 � �= P0
� 273 � T0
Pt P
or = 0 or P  T (if volume is kept constant)
T T0

At constant volume, the pressure of a given amount of a gas is directly proportional to its absolute
temperature.
Example 3: A certain amount of ethane is confined in a bulb of 1 liter capacity. The bulb is so weak that it
will burst if pressure exceeds 10 atm. Initially gas exerts 8 atm pressure at 270 C.
Find temperature at which the bulb will burst?
Solution: Considering limiting condition
P1 P
Since volume remain constant = 2
T1 T2
Thus 8/300=10/T2
 T2 = 375 K

2.5 Ideal Gas Law


The results of the laws of Boyle and Gas-Lussac can be combined into an expression which
represents the relationship between pressure, volume and temperature of a given mass of a gas;
such an expression is described as an equation of state.

Suppose the gas is in the initial state with volume, V 1, pressure p1and temperature T1. We then
change the state of the gas to a volume V 2, pressure p2 and temperature T2. Let us carry out this
change in two steps.
(i) First we change the pressure from p1 to p2 keeping the temperature T1 constant.The resultant
p1V1
volume Vr as given by Boyle’s law is Vr =
p2
(ii) Next, temperature is changed from T1 to T2, keeping the pressure p2 constant. The
final volume V2 as given by Charles law is

V2 =
Vr T2 ( p V /p ) T
= 1 1 2 2
p1V1 pV
= 2 2
or
T1 T1 T1 T2

CENTERS: MUMBAI / DELHI / AKOLA / KOLKATA / LUCKNOW / NASHIK / GOA #


GASEOUS ST AT E Rg - 2015 - 2017
It follows that no matter how we change the state of the given amount of a gas, the ratio pV/T
pV
always remains constant, i.e. =K
T

The value of K depends on the amount of gas in the system. Since V is an extensive property
(which is mass dependent), its value at constant p and T is proportional to the amount of the gas
present in the system. Then K must also be proportional to the amount of gas because p and T are
intensive properties (which have no mass dependence). We can express this by writing K = nR, in
which n is the amount of gas in a given volume of gas and R is independent of all variables and is,
therefore, a universal constant. We thus have the general gas law
pV = nRT
The universal gas constant R = pV/nT. Thus, it has the unit of (pressure �volume) divided by (amount
of gas �temperature). Now the dimensions of pressure and volume are,
Pressure = (force/area) = (force/length2) = force �length−2
Volume = length3
(force �length -2 ) (length 3 ) (force �length) work (or energy)
Thus R= = =
(amount of gas) (kelvin) (amount of gas) (kelvin) (amount of gas) (kelvin)

Thus, the dimensions of R are energy per mole per kelvin and hence it represents the amount of
work (or energy) that can be obtained from one mole of a gas when its temperature is raised by one
kelvin.

 Numerical Values of R

i) In liter atmosphere = 0.0821 litre atm deg–1 mole–1 ii) In ergs = 8.314  107 erg deg–1 mole–1

iii) In calories = 1.987 cal deg–1 mole–1 iv) In Joules = 8.314 J deg–1 mole–1

 Use the value of R depending on the units in which value of pressure and volume has been used
in ideal gas equation.

Example 4 : What mass of ammonia will exert same pressure as 12 g of H 2S(g) in the same container
under the similar conditions of temperature?
Solution: Under identical conditions of T and V, p  n
 equal moles of ammonia as that of H2S(g) will exert same pressure, when confined in
the same container
 Moles of H2S = 12/34 = moles of ammonia
 Mass of ammonia = (12/34)  17 = 6g

Example 5: 4 g of an ideal gas was confined in a 1.0 L flask at 1.0 atm. Increasing temperature of
flask by 30oC increases gas pressure by 8%. Determine molar mass of gas.
Solution: Let the initial temperature be, TK.
1 T
Since, n and V are constants P1/T1= P2/T2  =  T = 375K
1.08 T + 30
Since pV = nRT and n = w/M
CENTERS: MUMBAI / DELHI / AKOLA / KOLKATA / LUCKNOW / NASHIK / GOA #
GASEOUS ST AT E Rg - 2015 - 2017
4 11
 =  M = 123
M 0.082  375

 Relation between Molecular Mass and Gas Density

From the ideal gas equation

nRT w dRT d
P= = RT = M= RT
V M V M P

M= Molecular mass, P = Pressure, T=Temperature, d= Density

 Vapour Density
For gases another term which is often used is vapour-density. Vapour density of a gas is
defined as the ratio of the mass of the gas occupying a certain volume at a certain temperature and
pressure to the mass of hydrogen occupying the same volume at the same temperature and pressure
i.e.

W(gas) = PVM and


RT
PV  2
WH 2 = ( mol. wt.of Hydrogen is 2)
RT
Wgas M
 = = Vapour density of gas
WH2 2

 Vapour Density × 2 = Molecular wt.

 Vapour density of a gas is same at any temperature, pressure and volume.

Example 6: Determine the density of carbon dioxide gas at the sublimation temperature of -78oC
and 1.0 atm, assuming ideal behaviour of the gas.
pM 1  44
Solution:  = = = 2.75 g L-1
RT 0.082  195

Example 7: Determine payload of a 1000,000 L balloon filled with He gas at 27 oC and 1.0 atm.
Composition of air can be considered to be 79% N2 and 21% O2 by volume and balloon is
massless.
Solution: Moles of gas present in balloon
pV 1000,000
= = = 40650.4
RT 0.082  300

Payload = Wair - Wgas = 40650.4 (Mair - MHe) g

40650.4
= (28.84 - 4) kg = 1009.75 kg
1000

2.6 Dalton’s Law of Partial Pressure

CENTERS: MUMBAI / DELHI / AKOLA / KOLKATA / LUCKNOW / NASHIK / GOA #


GASEOUS ST AT E Rg - 2015 - 2017

The ideal gas equation pV = nRT applies to mixtures just as to pure gases. It was in fact with
a gas mixture, ordinary air, that Boyle, Gay-Lussac and Charles did their early experiments. The only
new concept we need in order to deal with gas mixtures is partial pressure.
The pressure exerted by a gas depends on the force exerted by each molecular collision with the
walls of the container and on the number of such collisions in a unit area per unit time. If a gas
contains two types of molecules, each species will engage in such collisions and thus, make a
contribution to the total pressure, in exact proportion to its abundance in the mixture. The
contribution that each species makes to the total pressure of gas is known as the “partial pressure” of
the species. Therefore, in other words, “partial pressure of a component gas, in a gaseous mixture, is
the pressure that would have been exerted, had the said component occupied the entire volume alone
under identical conditions of temperature.”

The statement of Dalton’s Law is “The total pressure of a mixture of non-reacting gases is equal to the
sum of their partial pressures”.

Let a mixture of gases have amount n1 of the first gas, n2 of the second gas, and so on. Let the corresponding
partial pressures be p1, p2, …… The total pressure is given by
ptotal = p1 + p2 + ….
If the gases present in the mixture behave ideally, then, it is possible to write separately for each gas,
p1V = n1 RT (a)
p2V = n2RT (b)
Hence (p1 + p2 + …) V = (n1 + n2 + …) RT
i.e., ptotal V = ntotalRT (c)
where ntotal is the total moles of gases in the mixture. Dividing Eqs (a) and (b) by Eq. (c), We get
n
p1 = 1 p total = x1p total
n total
n2
p2 = p total = x 2 p total
n total
x1 , x2, and so on are the mole fraction of each gas respectively

Partial Pressure of a gas = Mole fraction of the gas × Total Pressure of the gaseous mixture
 Dalton’s Law of Partial pressure is applicable only for non – reacting gases.

Example 8: Calculate partial pressure of nitrogen and oxygen in air assuming it to be composed of mostly
nitrogen and oxygen. Volume percentage of oxygen and nitrogen in air are 20 and 80
respectively, and atmospheric pressure to be 1.0.
Solution: Mole fraction of N2(g) = 0.8 and Mole fraction of O2(g) = 0.2
Partial pressure of N2(g) = 0.8  1 = 0.8 atm

CENTERS: MUMBAI / DELHI / AKOLA / KOLKATA / LUCKNOW / NASHIK / GOA #


GASEOUS ST AT E Rg - 2015 - 2017
Partial pressure of O2(g) = 0.2  1 = 0.2 atm
Example 9: 0.1 mol of ethane gas and 0.3 mol of oxygen gas are taken in a flask at 27 oC and 1.0 atm
pressure and sealed. Now the flask is heated to 1000 K where the following reaction
occurs quantitatively:
5
C2H6 + O2  2CO + 3H2O
2
Calculate partial pressure of each component at the end of reaction.
Solution: The balanced chemical reaction (with states specified) is
5
C2H6(g) + O2(g)  2CO(g) + 3H2O(g) at 1000 K
2
Moles at start: 0.10 0.3 0 0
Moles at end: 0 0.05 0.2 0.3
Total moles ng = 0.55
Now, applying gas laws at constant volume
p nT
1 = 11 �n T � �0.55 �1000 �
 p = p � 2 2 �= 1.0 � �= 4.58 atm
p n T 2 1 �n T �
2 2 2 �1 1 � �0.4 �300 �

�0.05 �
 p = � ��4.58 = 0.416 atm
O �0.55 �
2
 0.2 
p CO 2 =    4.58 = 1.664 atm and
 0.55 
�0.30 �
p = � ��4.58 = 2.5 atm
H O �0.55 �
2

Partial pressure and aqueous tension

Dalton’s law is used to calculate the pressure of a dry gas when it is


collected over water at atmospheric pressure. By Dalton’s law.

Pressure of dry gas = atmospheric pressure – aqueous tension

Aqueous tension is partial pressure of water vapour in air and it


depends only on temperature. It increases with temperature and
becomes 760 mm at 100°C.
2.7 Graham’s Law of Diffusion
If a tiny hole is made in the wall of a vessel containing a gas, then the rate at which gas
molecules leak out of vessel will be proportional to the number of molecules that collide with unit
area of the wall in unit time.
Diffusion is the tendency of any substance to spread throughout the space available to it.
Diffusion will take place in all direction and even against gravity. So gases diffuse through firm
substances and through small holes. The streaming of gas molecules through a small hole is called
effusion

CENTERS: MUMBAI / DELHI / AKOLA / KOLKATA / LUCKNOW / NASHIK / GOA #


GASEOUS ST AT E Rg - 2015 - 2017

Diffusion: mixing of gas molecules to Effusion: escape of a gas through a pinhole


minimize pressure gradient

Schottish physical chemist, Thomas Graham’s research on the diffusion of gases was triggered
by his reading about the observation of Germen chemist Johan Dobereiner that Hydrogen gas effused out of
a small crack in a glass bottle faster than the surrounding air diffused in to replace it. Graham measured the
rate of effusion of different gases experimentally through very fine tube. In this way he slowed down the
process, so that it could be studied quantitatively.
He plotted the following graph for complete effusion of 100 ml of different gases at 25oC.

According to Graham the rate of diffusion (effusion) of a gas at constant P&T is inversely proportional
to square root of its molecular mass.

1 r M
r� at constant P & T 1 = 2 at constant P & T
M r M
2 1

The above Equation is applicable only if two said gases are at same temperature
and pressure, effusing through openings of identical geometry. The two gases
may be in the combined system if they are non-reacting and at the same partial
pressure.

If a gaseous mixture consists of several components of different molar masses and at different partial
pressure as well, initial rate of effusion of a component will depend on both molar mass and their partial
pressure as
1
ri  pi and ri � M
i
p
i
 ri � , if rate of effusion is linearly related to its partial pressure.
M
i
r p M2
1 = 1
Under the above conditions,
r p M1
2 2
 Rate can be expressed in following terms

Volume diffused (V) moles diffused(n) distance travelled in a narrow tube(d)


r= = r=
time taken time taken time taken

CENTERS: MUMBAI / DELHI / AKOLA / KOLKATA / LUCKNOW / NASHIK / GOA #


GASEOUS ST AT E Rg - 2015 - 2017
Example 10: Rate of effusion of ethane is 1.53 times faster than rate of a hydrocarbon containing 14.27%
hydrogen by weight, under identical conditions. Deduce the molecular formula of
hydrocarbon.
r (ethane) M
Solution: = = 1.53 � M = 70.23
r ( hydrocarbo n) 30
Empirical formula C H
Wt.% 85.73 14.27
Mol % 85.73 / 12 14.27
SR 1 2
Empirical formula = CH2 ;

Molecular weight = 70.23


= Empirical formula weight  n = 14 n
n =5 ; hence, molecular formula of hydrocarbon is C5H10.

Example 11: Ammonia gas and HCl gas from the two flasks, at same temperature and pressure were
injected simultaneously through pinholes of similar geometry, attached at the two ends of a
1.0 m long glass tube. At what distance from the ammonia end, the first flash of white fume
would be observed?
Solution: Since, the two gases are injected at same pressure, their rate of effusion will depend only on
molar mass and ammonia being the lighter than HCl, will move at faster rate as illustrated
in the following diagrams:
100 cm
NH3 HCl
x cm

Point at which the two gases will meet first to


produce white fumes of NH4Cl

r( NH 3 ) x 36.5
= =  x = 59.43 cm
r( HCl ) 100 - x 17

Example12 : Diffusion of a certain volume of N2(g) at 1.0 atm and 300 K, takes 25s, while same volume of
an unknown gas of Xenon and Fluorine at 2.0 atm and 300 K takes 34 s for diffusion through
the same pinhole. Deduce the molecular formula of the unknown gas.
1 r( N2 ) 34 PN2 M 1 M
Solution: Rate �  = = =  M = 207.15
time r( gas ) 25 Pgas 28 2 28
Since atomic mass of Xe = 131, the gas cannot contain more than one Xe atom per
molecule.Hence, the molecular formula of unknown gas could be XeFn.
 207.15 = 131 + 19 n  x = 4 and gas is XeF4.

CENTERS: MUMBAI / DELHI / AKOLA / KOLKATA / LUCKNOW / NASHIK / GOA #


GASEOUS ST AT E Rg - 2015 - 2017
 Instantaneous rate of diffusion

Instantaneous rate of effusion will depend on the instantaneous partial pressure of that
component, which decreases continuously with progress of effusion. Considering, instantaneous rate of
decrease of partial pressure (−dp/dt) to be directly proportional to instantaneous gas pressure and inversely
proportional to square root of molar mass, instantaneous pressure at any time can be solved as
dp Kp
- = where K is constant of proportionality,
dt M
p2 t
dp K �p1 � Kt � Kt �
 �
- = �
dt  ln � �= or p2 = p1 exp �- �
p1
p M 0 �p 2 � M � M�

Hence, partial pressure decreases exponentially with time as

p ln p

t t

Example13: Pressure of nitrogen gas falls from 4000 mm to 2000 mm in 30 min, when allowed to effuse
through a pinhole in the cylinder. If the same cylinder is filled with an equimolar mixture of
N2 and He gas at 4000 mm of Hg, what would be the molar ratio of gases (N2 / He) in the
cylinder after 1.0 hour? Assume rate of decrease of pressure as linear function of gas
pressure.
Solution: Since, initially equal moles of gases are present, initial partial pressure of both N2 and He is
2000 mm of Hg. Also
dP KP  p1  Kt
- =  ln   = 1
dt M  p2  M
 4000  30K
For N2(g): ln  = … (i)
 2000  28
In mixture: For N2 :
 2000  60 K
ln  = … (ii)
 p( N )  28
 2 
 2000 
For He: ln   = 60 K … (iii)
 p  4
 ( Hg ) 
Solving Eqs. (i), (ii) and (iii), pN2 = 500 mm : pHe = 51 mm of Hg
Molar ratio after 1.0 hour (N2 : He) = p( N 2 ) : p( He ) = 500 : 51

 Diffusion in case of dissociation or association of gas


CENTERS: MUMBAI / DELHI / AKOLA / KOLKATA / LUCKNOW / NASHIK / GOA #
GASEOUS ST AT E Rg - 2015 - 2017
A polyatomic gas may remain in either pure form or it may remain in partially dissociated or associated
form. Dissociation or association alter the average molar mass of gas mixture compared to its pure form and
hence, their rates of effusion. To illustrate, let us consider one mole of PCl 5(g) confined in a flask. If it is
considered to be 100% pure, its mass will be 208.5 g. In actual practice, PCl 5(g) always remains in
equilibrium with some of its dissociation products

���
PCl5 (g) ��� PCl3 (g) + Cl 2 (g) Total mole = 1 + α > 1
1-a a a

Due to the above dissociation, moles have changed from 1.0 to (1 + α), but the mass remains same,
208.5 g.

Weight of mixture 208.5


Now, molar mass of mixture = = < 208.5
Total moles of mixture 1 + a

Hence, in case of dissociation, average molar mass of mixture decreases compared to molar mass of pure
gas, therefore, rate of effusion of partially decomposed gas mixture is always greater than the rate of
effusion of a pure gas.

In case of association, number of moles decreases and hence, molar mass of partially associated gaseous
mixture is always greater than molar mass of pure gas as

2NO 2 ���
��� N 2O 4 α
Total mole = = 1 - < 1  Mmix > M ( NO2 )
1-a a /2 2

Thus, if a gas undergoes partial association, its rate of effusion will be less than the rate of effusion of pure
gas (without considering any association).

Example 14: Rate of effusion of ethane gas is 1.9 times, the rate of effusion of a partially decomposed
Cl2O7 (g) mixture. Determine the degree of dissociation of Cl2O7 (g).
r( C2 H 6 ) M mix M ( Cl 2O7 ) 183
Solution: = = 1.9  M mix = 108.3 = =  a =
rmix 30 1 + 3.5 a 1 + 3.5 a
0.197

 Separation of gases using diffusion

When a gaseous mixture containing lighter and heavier components is allowed to pass through several
diffusion chambers connected in series, there occur enrichment of lighter component in each successive
step. By carrying out diffusion of a gaseous mixture for a specific number of steps in succession, a specified
enrichment of lighter component can be achieved as follows:
1 2 3 4 n

A, B

�rA � �n A � M B �n �
n: � �= � � = �A �
�rB �
1 �n B �
1
MA �n B �
2

CENTERS: MUMBAI / DELHI / AKOLA / KOLKATA / LUCKNOW / NASHIK / GOA #


GASEOUS ST AT E Rg - 2015 - 2017
2/2
�rA � �n A � M B �n � �M � �n �
� �= � � = �A � � B � = �A �
nd
From the 2 chamber:
�rB �
2 �n B �
2
MA �n B � M
1 � A � �n B �
3

3/ 2
�rA � �n A � M B �n � �M � �n �
� �= � � = �A � � B � = �A �
rd
From the 3 chamber:
�rB �
3 �n B �
3
MA �n B � M
1 � A � �n B �
4

n-1
� � � � �n � �M �2 �n �
Similarly, from (n-1) chamber: �rA � = �n A �
th MB
= �A � � B � = �A �
�rB �
n-1 �n B �
n-1
MA �n B � M
1 � A � �n B �
n

n-1
� � � � � �2 �n �
 �rA � = �n A � �M B � = �A �
�rB �
n-1 �n B � M
1 � A � �n B �
n

This Equation indicates that after (n -1)th step of diffusion a specific enrichment (nA/nB)n can be achieved in
the nth chamber.

Example 15: A sample of Ne is originally 10% by mole in Ne20 isotope and remaining are Ne22 isotope. In
how many steps of effusion, 25% enrichment of Ne20 can be achieved?
 Ne 20  10 1
Solution: Initially  22
 = =
 Ne  90 9

 Ne 20  25 1
Desired ratio :  22
 = =
 Ne  75 3
n -1
n -1 �22 �
Applying equation : 1  22  1 log � �= log 3 � n = 24
2
= �
9  20  3 2 �20 �

2.8 Eudiometry
Eudiometry or “gas analysis” involves calculations based on gaseous reactions in which at least two
components are gases & their amount is given in terms of volumes measured at same pressure &
Temperature. The relationship amongst gases, when they react with one another, is governed by two laws,
namely Gay-Lussac law and Avogadro’s law.
 Avogadro’s Law
In 1812, Amedeo Avogadro stated that samples of different gases which contain the same
number of molecules (any complexity, size, shape) occupy the same volume at the same temperature
and pressure. It follows from Avogadro’s hypothesis that V  n (when T and P are constant).
Gaseous reactions for investigation purposes are studied in a closed graduated tube open at one end
and the other closed end of which is provided with platinum terminals for the passage of electricity
through the mixture of gases. Such a tube is known as Eudiometer tube and hence the name
Eudiometry also used for Gas analysis.
During Gas analysis, the Eudiometer tube filled with mercury is inverted over a trough
containing mercury. A known volume of the gas or gaseous mixture to be studied is next introduced,
which displaces an equivalent amount of mercury. Next a known excess of oxygen is introduced and
the electric spark is passed, whereby the combustible material gets oxidised. The volumes of carbon
CENTERS: MUMBAI / DELHI / AKOLA / KOLKATA / LUCKNOW / NASHIK / GOA #
GASEOUS ST AT E Rg - 2015 - 2017
dioxide, water vapour or other gaseous products of combustion are next determined by absorbing
them in suitable reagents. For example, the volume of CO 2 is determined by absorption in KOH
solution and that of excess of oxygen in an alkaline solution of pyrogallol. Water vapour produced
during the reaction can be determined by noting contraction in volume caused due to cooling, as by
cooling the steam formed during combustion forms liquid (water) which occupies a negligible
volume as compared to the volumes of the gases considered. The excess of oxygen left after the
combustion is also determined by difference if other gases formed during combustion have already
been determined. From the data thus collected a number of useful conclusions regarding reactions
amongst gases can be drawn.
a) Volume-volume relationship amongst Gases or simple Gaseous reactions.
b) Composition of Gaseous mixtures.
c) Molecular formulae of Gases.
d) Molecular formulae of Gaseous Hydrocarbons.

The various reagents used for absorbing different gases are


O3  turpentine oil
O2 alkaline pyrogallol
NO  FeSO4 solution
Cl2, CO2,SO2 alkali solution (NaOH, KOH, Ca(OH)2, HOCH2CH2NH2, etc.)
NH 3 � Water
H 2O � CuSO4 , CaCl2
CO � Ammonical Cu2Cl2
General Assumptions: In all problems, it is assumed that the sparking occurs at room temperature. This
implies that water formed would be in liquid state and that nitrogen gas is inert towards oxidation. Also,
if a gas is added externally, it will be assumed to be in excess unless otherwise stated.

 Determination of Molecular formula of Hydrocarbon using Eudiometry


A known amount of hydrocarbon is taken into an Eudiometry tube. O 2 gas is then inserted to cause complete
combustion of hydrocarbon & the reaction mixture is cooled back to the original room temperature. This
gives 1st volume contraction VIC. The resultant gaseous mixture is then passed through alc.. KOH which
gives second volume contraction VII c. These data can help to calculate the molecular formula of the
hydrocarbon as explained below.
Step 1:Write down the balanced chemical reaction.
� y� y
C x H y + �x + �
O2 � xCO2 + H2O
� 4� 2
Step 2:Write down Volume of components before the reaction
VHC VO2 0 -
Step 3:Write down volume of components after the reaction using Avogadro’s Law,
(after identifying limiting Reagent)
� y�
0 VO2 - �x + �
VHC xVHC -
� 4�
CENTERS: MUMBAI / DELHI / AKOLA / KOLKATA / LUCKNOW / NASHIK / GOA #
GASEOUS ST AT E Rg - 2015 - 2017
Step 4:Use the data given
VIC = Vbefore – Vafter [due to change in volume because of reaction]
� � y� � � y�
= VHC + VO2 - �
VO2 - �x + �
VHC + xVHC �= VHC �
1+
� � 4� � � 4�

This will give the value of y

VIIC = due to change in volume because of absorption of CO2 in alc KOH.


�VIIC = xVHC
Hence, both x & y can be calculated.

Example 16: A gaseous hydrocarbon requires 6 times its own volume of O2 for complete oxidation
and produces 4 times its volume of CO2. What is its formula?

Solution: The balanced equation for combustion


� y� y
C x H y + �x + �
O2 � xCO2 + H 2 O
� 4� 2
� y� y
1 vol. �x + �vol x + = 6 , or 4x + y = 24.............. ( 1)
� 4� 4
Again x = 4 since evolved CO2 is 4 times that of hydrocarbon
 16 + y = 24 or y = 8  formula of hydrocarbon C4H8

3. KINETIC THEORY OF GASES


All gaseous laws (e.g., Boyle’s law, Charles’ law etc.) are concise statements of experimental facts
observed in the laboratory by the scientists. Conducting careful experiments is an important aspect of
scientific method and it tells us how the particular system is behaving under different conditions. However,
once the experimental facts are established, a scientist is curious to know why the system is behaving in that
way. A theory is constructed to answer this question. A theory is a model (i.e., a mental picture) that enables
us to better understand our observations. The theory that attempts to elucidate the behaviour of gases is
known as kinetic molecular theory.
In order to derive the theoretical aspect of the various gas laws based on simple experimental facts, Maxwell
proposed the following postulates under the heading of kinetic theory of gases.

3.1 Postulates of KTG


 A gas consists of a large number of very small spherical tiny particles, which may be identified as
molecules. The molecules of a given gas are completely identical in size, shape and mass.
 The volume occupied by the molecules is negligible in comparison to the total volume occupied by the
gas (i.e. volume of the container).
 The molecules are in rapid motion which is completely random. During their motion, they collide with
one other and with the sides of the vessel. The pressure of the gas is due to the collisions of molecules
with the sides of the vessel.

CENTERS: MUMBAI / DELHI / AKOLA / KOLKATA / LUCKNOW / NASHIK / GOA #


GASEOUS ST AT E Rg - 2015 - 2017
 The molecules are perfectly elastic, i.e. there occurs no loss of energy when they collide with one
another and with the sides of the vessel.
 The laws of classical mechanics, in particular Newton’s second law of motion, are applicable to the
molecules in motion.
 There is no force of attraction or repulsion amongst the molecules, i.e. they are moving independent of
one another.
 At any instant, a given molecule can have kinetic energy ranging from a small value to a very large
value, but the average kinetic energy remains constant for a given temperature, i.e. the average kinetic
energy is proportional to the absolute temperature of the gas.

3.2 Velocity Distribution of Gas molecules


Speed of a gas molecule changes continuously due to the intermolecular collisions and their collisions with
the wall of container. Thus, speed of an individual molecule is not constant. Also, the observable properties
of gas such as volume, pressure and temperature are constant with time. It is expected to be applicable to the
distribution of molecular speed and we consider a statistical average of the speed of whole of the collection
of gas molecules to remain constant with time. That is the fraction of the total molecules having speed
between any definite ranges must be constant, even though the speeds of the individual molecules may be
changing as a result of molecular collisions. Distribution of molecular speed over a possible range was first
investigated by Maxwell using the theory of probability. Results were expressed as the Maxwell law for
distribution of molecular speed as
3/ 2 - mu 2
�M �
dN u = 4p N � � e
2RT
u 2 du
�2p RT �
3/ 2 - mu 2
�m �
= 4p N � � e
2kT
u 2 du
�2p kT �
The above expression gives the number of molecules dN u having speeds
between u and u + du in term of total number of molecules present (N), Fraction
of the
molar mass (M) of gas and temperature (T). According to expression the Molecule

fraction of molecules (dNu/N) having the speeds in the range of u and u +


du, for a gas of molar mass (M) depends only on temperature. Hence, for a Speed

given temperature, this fraction is constant. A plot of fraction of molecules in the speed range u and u + du,
1 �dNu �
� �
N �du �vs u is described in the graph.

The peaks in the curve correspond to a speed, which is possessed by maximum fraction of molecules, called
“most probable speed”.

CENTERS: MUMBAI / DELHI / AKOLA / KOLKATA / LUCKNOW / NASHIK / GOA #


GASEOUS ST AT E Rg - 2015 - 2017
 Some conclusive points for distribution of molecular speeds are :

(i) The fraction of molecules having either very high or very low speeds is very small. Majority of
molecules have speed near to umps in the middle of the range of molecular speeds.

(ii) The total area under the curve in Figure is a measure of total number of molecules in collection

(iii) Since, rise in temperature raises the kinetic energy of gas


molecules, it follows that fractions of molecules having T1
Fraction
of the T2
lower speed range decreases and fraction of molecules Molecule

having higher speed range increases. Also the curve at


higher temperature has its umps shifted to higher value but Speed
For a given gas and number of mole but diffrent temperature
the corresponding fractions of molecules have decreased.
Thus in general, the distribution of speeds is wider at
higher temperature than at lower temperature as shown in
X
figure (T2>T1) Fraction
Y
of the
Molecule

(iv) Distribution of molecular speed also depends upon the


Speed
mass of gas molecule. At similar temperature, a heavier For a given number of mole and temperature but diffrent gas

gas molecule has a narrow distribution of speeds than a lighter gas molecule as shown in the
following diagram (MX>MY):

As per kinetic theory of gases, each molecule is moving with altogether different velocity. Let ‘n’
molecules be present in a given mass of gas, in which n 1 molecules have a speed of v1, n2 molecules
have a speed of v2, ,nn molecules have a speed of vn.
The average velocity or Uav = average of all such velocity terms.
n1u1 + n2u2 + n3u3 + ........ + nnun
Average velocity =
n
8RT
Using Maxwell distribution average velocity is determined as Uav =
πM
Another mean velocity proposed by Maxwell was U rms as the square root of means of square of all
such
velocities.

n1u12 + n2u22 + n3u32 + ........+ nnun2


U rms =
n

3RT
Using Maxwell distribution rms velocity is determined as Urms =
M

CENTERS: MUMBAI / DELHI / AKOLA / KOLKATA / LUCKNOW / NASHIK / GOA #


GASEOUS ST AT E Rg - 2015 - 2017

2RT
Most probable speed is the velocity which is possessed by maximum no. of molecules. Vmp =
M
2RT 8RT 3RT 8
Furthermore Ump : Uav : Urms : : : : = 2 : : 3 =1 : 1.128 : 1.224
M πM M π
Also Uav = Urms  0.9213

 For calculating Vrms, Vmp, or Vav by above relations the value of M should be used in kg/mole
and R should be taken as 8.314 J/K mole to get velocity in m/s.

Example 17: Derive an expression relating to increase in U rms of a gas for a relatively small
temperature rise and calculate increase in urms for a sample of Ne(g) as the temperature is
increased from 100 K to 101 K?
3RT durms 3R 1 3R 1 3R
urms = = = =  =
Solution: M dT M 2 T 2M 3RT 2 Murms
M
3R 3R
� du rms = dT � Du rms = DT
2Mu rms 2Mu rms
3RT 3  8.314  100
u rms = = = 353 ms-1
M 20  10 -3
3  8.134
Du rms =  (101 - 100) = 1.766 ms-1
2  20  10-3  353

3.3 Kinetic interpretation of Pressure


Imagine a cube of edge-length l, containing N molecules, each having a mass of m. Molecules are moving at
random in all directions, with speed covering a considerable range of
values. G C

The velocity u1 of any molecule may be resolved into three-component


H
velocities designated as ux, uy and uz.. These are in the three directions at D

right angles to each other and parallel to the sides of the cube as shown in
uz
figure. The component velocities are related by the expression u
ux
u12 = u 2x + u 2y + u 2z … (1) uy
F B
Considering the x-component motion of a molecule, we will have
Momentum of molecule before collision with the side ABCD = mux.
E
Momentum of the molecule after collision with the side ABCD = − mux. l A

Change of momentum of the molecule in a single collision with the side ABCD = |2mux|.
Since l is the edge length of the cube, the molecule has to travel a distance 2l to arrive back at the wall
ABCD. The number of collisions per unit time with the wall ABCD will be equal to ux /2l.
2
�u � mu x
The total change of momentum per unit time due to such impacts is 2mu x � x �=
�2l � l
According to Newton’s second law of motion

CENTERS: MUMBAI / DELHI / AKOLA / KOLKATA / LUCKNOW / NASHIK / GOA #


GASEOUS ST AT E Rg - 2015 - 2017
d (velocity) d
Force = mass x acceleration= mass  = (mass  velocity)
dt dt
d
= (momentum) = rate of change of momentum
dt
2
Hence, total force due to impacts of a single molecule with the wall ABCD of the vessel is mu x / l. The area
of the wall is l2. Hence, the pressure exerted due to the collision of x-component velocity of a single
molecule with the side ABCD is
mu 2x /l mu x2
px = = where V is the volume of the vessel.
l2 V
Since each molecule will exert similar pressure, the total pressure exerted on the wall ABCD will be
N
M N 2
p= �p
i=1
ix = �u x
V i=1
N
1 mN 2
Defining the mean square speed as u 2x =
N
�u
i =1
2
ix we can write p=
V
ux (2)

Since the directions x, y and z are equivalent, we will also have u 2x = u 2y = u 2z (3)

But from Eq. (1), we will have u 2 = u 2x + u 2y + u 2z

2 2 2 1 2
From Eqs (1) and (3), we can write u x = u y = u z = u
3
mN �1 2 � 1
Substituting this in Eq. (2), we get p = � u � or pV= mNu 2
V �3 � 3
3RT
From Maxwell distribution we already know u =
2

M
1 3RT RT
So pV = mN  = nM  = nRT � pV=nRT
3 M M

 Average Translational kinetic energy per molecule = ½ murms2 = 3/2kT


 Average Translational Kinetic energy per mole = NA × ½ murms2 = 3/2 RT
 k(Boltzmann constant) = R/NA = 1.38 × 10-23 JK-1

Example 18: Calculate the pressure exerted by 1025 gas molecules each of mass 10 -22 g in a container of
volume 1.0 dm3. The root mean square speed is 105 cm s-1.
1 1
 10 -25 kg  1025  (103 ms -1 ) 2 
1 1
Solution: pV = mN u 2  p= ޴ 109 Pa
3 3 10-3 m 3 3
Example 19: 33
pV 106 �2 �10 -3 �6.022 �1023
Solution: pV = nRT � T= = = 483K
nR 3 �1023 �8.314
3 3
� E trans = k b T = �1.38 �10-23 �483 = 10 -20 J / moleule
2 2

CENTERS: MUMBAI / DELHI / AKOLA / KOLKATA / LUCKNOW / NASHIK / GOA #


GASEOUS ST AT E Rg - 2015 - 2017
3
Etrans (Total ) = kb TN = 3000.25 J
2

4. REAL GASES
The ideal gas laws are derived from the kinetic theory of gases which is based on the following two
important assumptions:
(i) The volume occupied by the molecules is negligible in comparison to the total volume of the gas.
(ii) The molecules exert no forces of attraction upon one another.
It is because neither of these assumptions can be regarded as applicable to real gases that the real
gases show departure from the ideal behaviour.

4.1 Van der waal Equation


Van der Waal was the first to introduce systematically the correction
2r
terms due to the above two invalid assumptions in the ideal gas excluded
equation PV = nRT. His corrections are given below. volume

 Volume Correction

V in the ideal gas equation represents an volume where the molecules


can move freely. In real gases, a part of the total volume is, however,
occupied by the molecules of the gas.. If b represents the effective
volume occupied by the molecules of 1 mole of a gas, then for the amount n moles of the gas V i is given by
V = V container - nb
where b is called the excluded volume or co-volume. The numerical value of b is four times the actual
volume occupied by the gas molecules. This can be shown as follows. If we consider only bimolecular
collisions, then the volume occupied by the sphere of radius 2r represents the excluded volume per pair of
molecules as shown in Fig
4 �4 �
Thus excluded volume per pair of molecules π(2r)3 = 8 � πr 3 �
3 �3 �
1 � �4 3�
� �4 3 �
Excluded volume per molecule = �8πr
� =�
4
� πr
� =� 4 (volume occupied by a molecule)
2 � �3 �
� �3 �
� �4 3 �

Since b represents excluded volume per mole of the gas, it is obvious that b = N A �
4πr
� �

� �3 �

 Pressure Correction

CENTERS: MUMBAI / DELHI / AKOLA / KOLKATA / LUCKNOW / NASHIK / GOA #


GASEOUS ST AT E Rg - 2015 - 2017
Consider a molecule A in the bulk of a vessel as shown in fig. This molecule
is surrounded by other molecules in symmetrical manner, with the result that
this molecule on the whole experiences no net force of attraction.
Now, consider a molecule B near the side of the vessel, which is about to
strike one of its sides, thus contributing towards the total pressure of the gas.
There are gas molecules only on one side of the vessel, i.e. towards its centre,
with the result that this molecule experiences a net force of attraction towards
the centre of the vessel. This results in decreasing the velocity of the molecule, and hence its
momentum. Thus, the molecule does not contribute as much force as it would have, had there been no
force of attraction. Thus, the pressure of a real gas would be smaller than the corresponding pressure of
an ideal gas, i.e. pi = pr + correction term

This correction term depends upon two factors:


(i) The number of molecules per unit volume of the vessel :- larger this number, larger will be the force of
attraction with which the molecule B is dragged behind. This results in a greater decrease in the
velocity of the molecule B and hence a greater decrease in the rate of change of momentum.
Consequently, the correction term also has a large value. If n is the amount of the gas present in the
volume V of the container, the number of molecules per unit volume of the container is given as
nN A n
N�
= or N��
V V
n
Thus, Correction term 
V
(ii) The number of molecules striking the side of the vessel per unit time:- Larger this number, larger will
be the decrease in the rate of change of momentum. Consequently, the correction term also has a larger
value. Now, the number of molecules striking the side of vessel in a unit time also depends upon the
number of molecules present in unit volume of the container, and hence in the present case:
n
Correction term 
V
Taking both these factors together, we have
�n ��n � 2
n2
Correction term  � �� � or Correction term a n 2 � correction term = a
�V ��V � V V2
Where a is the proportionality constant and is a measure of the forces of attraction between the
n2
molecules. Thus pi = pr + a
V2
When these expressions are substituted in the ideal gas equation pi Vi = nRT, we get
� n2a �
�p + 2 �(V - nb) = nRT
� V �
This equation is applicable to real gases and is known as the Van der Waals equation.

CENTERS: MUMBAI / DELHI / AKOLA / KOLKATA / LUCKNOW / NASHIK / GOA #


GASEOUS ST AT E Rg - 2015 - 2017
 The constants a & b: Van der Waals constant for attraction (a) and excluded volume (b) are
characteristic for a given gas. Some salient features of a & b are:
i) For a given gas Vander Waal’s constant of attraction ‘a’ is always greater than Vander Waals
constant of excluded volume (b).
ii) The gas having higher value of ‘a’ can be liquefied easily and therefore H2 & He are not liquefied
easily.
iii) The units of a = litre2 atm mole–2 & that of b = litre mole –1
iv) The numerical values of a & b are in the order of 10–1 to 10–2 & 10–2 to 10–4 respectively.
v) Volume correction factor, depends on molecular size and larger molecule will have larger b.
For example, size of He, CH4, CF4, C4H10 are in order of He < CH4 < CF4 < C4H10 and same will be
the order of b.
n 2a
vi) Pressure correction factor ( ) depends on intermolecular force of attraction. Hence, larger the
V2
intermolecular force of attraction larger the value of ‘a’, for same n and V. For example,
intermolecular force of attraction among the molecules H2, CO2, NH3 are in order of
H2 < CO2 < NH3 (H-bonding) thus same is the order of a.

4.2 Compressibility Factor


The deviations from ideal behavior can be displayed more clearly, by plotting the ratio of the
observed molar volume Vm to the ideal molar volume Vm,ideal (=RT/p) as a function of pressure at constant
temperature. This ratio is called the compressibility factor Z and can be expressed as
Vm p
Z= = Vm
Vm,ideal RT

For an ideal gas Z=1 and is independent of pressure and temperature. For a real gas,
Z = (T, p), is a function of both temperature and pressure.

i) At low pressures: ‘V’ is large and ‘b’ is negligible in comparison with V. The Vander Waals equation
reduces to:
� a � a
�P+ 2 �V=RT ; PV + = RT
� V � V
a
PV = RT - or PV < RT
V

This accounts for the dip in PV vs P isotherm at low pressures.

ii) At fairly high pressures


a
may be neglected in comparison with P. The Vander Waals equation becomes
V2
P (V – b) = RT
PV – Pb = RT

CENTERS: MUMBAI / DELHI / AKOLA / KOLKATA / LUCKNOW / NASHIK / GOA #


GASEOUS ST AT E Rg - 2015 - 2017
a
iii) At very low pressures: V becomes so large that both b and become negligible and the Vander
V2
Waals equation reduces to PV = RT. This shows why gases approach ideal behaviour at very low
pressures.
iv) Hydrogen and Helium: These are two lightest gases known. Their molecules have very small masses.
a
The attractive forces between such molecules will be extensively small. So is negligible even at
V2
ordinary temperatures. Thus PV  RT.
0°C

2 CO2 50° C
100°C
N2
100°C
H2 Z
He
Ideal gas
1.0
Z 1 50°C
0°C

0 P P (atm)

(Deviation of gases from ideal The plot of Z vs P for N2 gas at different temperature is
behaviour with pressure.) shown here.
Example20: The density of steam at 100oC and 1.0 atm pressure is 0.5974 kg m -3. Determine
compressibility factor for steam in the given condition.
Solution Since Z=Vm,real/Vm,ideal = ρideal/ρreal
pM 1  18
Z = = = 0.985
RT 0.5974  0.082  373

4.3 Liquefaction of Gases


First complete data on pressure - volume -temperature relations of a substance in both gaseous and
liquid state was obtained by Thomas Andrews on carbon dioxide. He plotted isotherms of carbon dioxide at
various temperatures. Later on it was found that real gases behave in the same manner as carbon dioxide.
Andrews noticed that at high temperatures isotherms look like that of an ideal gas and the gas cannot be
liquefied even at very high pressure. As the temperature is
lowered, shape of the curve changes and data shows
considerable deviation from ideal behaviour. At
30.98 °C carbon dioxide remains gas upto 73 atm
pressures. At 73 atm pressure, liquid carbon dioxide
Pressure

appears for the first time. The temperature 30.98 °C is


called critical temperature (TC) of carbon dioxide.
The critical temperature, pressure and volume are called
critical constants. Further increase in pressure simply
compresses the liquid carbon dioxide and the curve
represents the compressibility of the liquid. The steep line
represents the isotherm of liquid. Even a slight
Volume
compression results in steep rise in pressure indicating
very low compressibility of the liquid. Below 30.98 °C, the behavior of the gas on compression is quite

CENTERS: MUMBAI / DELHI / AKOLA / KOLKATA / LUCKNOW / NASHIK / GOA #


GASEOUS ST AT E Rg - 2015 - 2017
different. At 21.5 °C, carbon dioxide remains as a gas only upto point B. At point B, liquid of a particular
volume appears. Further compression does not change the pressure. Liquid and gaseous carbon dioxide
coexist and further application of pressure results in the condensation of more gas until the point C is
reached. At point C, all the gas has been condensed and further application of pressure merely compresses
the liquid as shown by steep line. Below 30.98°C each curve shows the similar trend. Only length of the
horizontal line increases at lower temperatures. At critical point horizontal portion of the isotherm merges
into one point.
��P� ��2 P �
So at point E � � = 0 and � 2 �=0
��
V�TC ��V � T C

Critical temperature Tc is the maximum temperature at which a gas can be liquefied, i.e. the temperature
above which a liquid cannot exist.
Critical pressure pc is the minimum pressure required to cause liquefaction at the temperature Tc.
Critical volume Vc is the volume occupied by one mole of a gas at critical temperature T c and critical
pressure pc. The point E in Fig represents the gas in its critical state. At this point the temperature, pressure
and volume have critical values. These three are known as critical constants. The isotherm corresponding to
the temperature Tc is known as critical isotherm. To the left of the point E on this isotherm, we have the
liquid state whereas to the right, we have the gaseous state. Thus, at point E transition from liquid to gaseous
state (or vice versa) takes place and thus it is not possible to state whether the substance is in the gaseous
form or in the liquid form. In fact, both the states become indistinguishable at the critical point. The surface
of separation between liquid and gas disappears. At this point the various physical properties such as density,
refractive index, etc., have identical values for both the states.
��p� ����p ��
� � = 0 and the condition that this slope has a maximum value of �
� �� = 0
��
V�T �
V���V�T T

RT a
From the van der Waals equation p = - 2 we get
Vm - b Vm
 p  RT 2a
  = - + 3
 Vm T (Vm - b ) Vm
2

��2 p � 2RT 6a
� 2 �= - 4 .
( Vm - b )
3
��Vm �T
Vm

RTc 2a
Hence at the critical point, - + =0
( Vc - b )
2
Vc3

2RTc 6a
and - + =0.
( Vc - b )
3
Vc4

8a
Solving Eqs. for Vc and Tc, we get Vc = 3b, Tc =
27Rb
Substituting the values of Vc and Tc in the van der Waals equation, we have
RTc a R(8a/27Rb) a 4a a a
pc = - 2 = - = - =
Vc - b Vc (3b - b) (3b) 2 27b 2
9b 2
27b 2
4.4 Virial equation

CENTERS: MUMBAI / DELHI / AKOLA / KOLKATA / LUCKNOW / NASHIK / GOA #


GASEOUS ST AT E Rg - 2015 - 2017

All real gas equations of state can be expressed approximately in one common form, called the Virial
equation of state which has the following form for 1 mole of a gas.
pVm 1 1 1
Z= =1+ B + C 2 + D 3 +....
RT Vm Vm Vm

where B, C, …. are temperature dependent constants known as second, third, etc., virial coefficients. These
coefficients must be evaluated experimentally at each different temperature.
The van der Waals equation of state for 1mole of a gas is

� a � RT a
�p + 2 �(Vm - b) = RT or p= - 2
� Vm � Vm - b Vm
-1
pVm Vm a � b � a
Multiplying both sides by Vm/RT, we get RT = V - b - V RT 1-
or Z = � �-
m m � Vm � Vm RT
-1
In the low pressure region, Vm is large and b/Vm << 1. Thus, the expression(1 – b/V m) can be expanded into
a power series in b/Vm:

-1 2 2
� b � b �b � �b �
1-
� � =1 + + � � + � � + ...
� Vm � Vm �Vm � �Vm �
2
� a �1 �b �
b-
Substituting this in the expression for Z, we get Z = 1 + � � + � � + ...
� RT �Vm �Vm �
a
Thus for the second virial coefficient, we have B = b -
RT
2
Third virial coefficient C = b , and so on.
An alternate form of the virial equation of state involves the expression of Z in terms of a power series in p,
2
i.e. Z = 1 + A1p + A2p + …

 BOYLE’S TEMPERATURE
1 � a � b2
Also the expansion series Z = 1 + b
� - �+ 2 + ... is a decreasing
Vm � RT � V m
1 � a �
series and can be approximated as: Z=1+ �b - �
Vm � RT �
1 � a �
Imposing the condition of ideality: Z = 1 � �b - �= 0
Vm � RT �

CENTERS: MUMBAI / DELHI / AKOLA / KOLKATA / LUCKNOW / NASHIK / GOA #


GASEOUS ST AT E Rg - 2015 - 2017
a
 T= =Boyle’s temperature (TB)
Rb
Boyle temperature is that temperature at which a real gas behaves like an ideal gas for a range of pressure.
Example 21: Calculate the pressure exerted by a mole of CO 2 gas confined in a 0.095 L container at 100oC.
The 1st and 2nd coefficients Bv RT and Cv RT are -2.26 (l2 atm mol-1) and 0.161 (L3 atm mol-1)
respectively.
Solution: Virial equation in volume is:
pVm B C
= 1 + v + v2 + ....
RT Vm Vm
RT  RT  C RT
 p= + Bv  2  + v 3
Vm  Vm  Vm
0.0821  373 - 2.26 0.161
= + 2
+ = 259.32 atm
0.095 (0.095) (0.095) 3

5.1 SOLVED PROBLEMS (OBJECTIVE)


Example 1 Which of the following curve does not represent Boyle’s law?

(B) log P

log V

(
C )P (D) P

1
V V
C 1
Solution: P= Where C is a constant. We can see that (c) is true as the graph of P vs would be a
V V
straight line.
(B) is true because log P = log C – log V.
dP - C
(A) is true because = 2
dV V
which means that as V increases the slope decreases and is always negative  (D)

Example 2 Boyle’s law may be expressed as

(A)  dP  = K (B)  dP  = - K2 (C)  dP  = - K (D) none


 dV  T V  dV  T V  dV  T V

CENTERS: MUMBAI / DELHI / AKOLA / KOLKATA / LUCKNOW / NASHIK / GOA #


GASEOUS ST AT E Rg - 2015 - 2017
 dP  P K
Solution: from Boyle’s law; PV = constant; PdV +VdP = 0;   =- =- 2
 dV  T V V
Thus (PV = K)  (B)

Example 3 A commercial gas cylinder contains 75 L of He at 15 bar (Gauge pressure). Assuming ideal
gas behavior for the isothermal expansion, how many 3.0 L balloons at 1.1 bar pressure can
be filled by the gas in the cylinder?
(A) 338 (B) 430 (C) 403 (D) 304
Solution: Assuming atmospheric pressure to be one bar, initial pressure and final pressure
of He gas present in cylinder will be:
pi = 15 + 1 = 16 bar and pf = 1.1 bar
16  75
Volume of He gas when expanded isothermally to 1.1 bar = = 1090.9L
1.1
Out of 1090.9L, 75 L of gas will remain in cylinder since this point pressure
equilibrium will be established.
1090.9 - 75
� Numbers of balloons = = 338 Thus (A)
3
Example 4 A vessel has N2 gas saturated with water vapor at a total pressure of 1 atm. The partial
pressure of water vapour is 0.3 atm. The contents of this vessel are transferred to another
vessel having one third of the capacity of original volume, at the same temperature the total
pressure of this system in the new vessel is
(A) 3.0 atm (B) 1 atm (C) 3.33 atm (D) 2.4 atm
Solution: PN' 2 + PH' 2O = 1atm PH' 2O = 0.3 atm PN' 2 = 0.7 atm

Now new pressure of N 2 in another vessel of volume V/3 at same temperature T is given by

" V1
PN  = 0.7 V 
2
3

PN" 2 = 2.1 atm


since aqueous tension remains constant, and thus total pressure in new vessel
" '
= PN 2
+ PH2 O
= 2.1 + 0.3 = 2.4 atm  (D)

Example 5 X ml of H2 gas effuses through a hole in a container in 5 seconds. The time taken for the
effusion of the same volume of the gas specified below under identical conditions is:
(A) 10 seconds: He (B) 20 seconds: O2
(C) 25 seconds: CO (D) 35 seconds: CO2
rH 2 4 rH 2 32
Solution: = = 2  (A) is incorrect ; = =4 (B) is correct
rHe 2 rO2 2

CENTERS: MUMBAI / DELHI / AKOLA / KOLKATA / LUCKNOW / NASHIK / GOA #


GASEOUS ST AT E Rg - 2015 - 2017

rH 2 28 rH 2 2 1
= = 14 (C) is incorrect ; = = (D)is incorrect  (B)
rCO 2 rCO2 44 12

Example 6 In what molar ratio He and CH 4 should be mixed so that when the mixture is allowed to
effuse through a pinhole, initially both gases come out at equal rate?
(A) 2:1 (B) 1:1
(C) 2:3 (D) 4:1
n
Solution: Since, rate of effusion ( r ) = from a mixture.
M
r ( He) n( He) 16
= = 1.0 � n CH4 : n He = 2 :1  (A)
r (CH 4 ) n(CH 4 ) 4
Example 7 Dalton’s law of partial pressure is not applicable to, at normal conditions
(A) H2 and N2 mixture (B) H2 and Cl2 mixture
(C) H2 and CO2 mixture (D) H2 and O2 mixture
Solution: H2 and Cl2 reacts to form HCl; Dalton’s law of partial pressure is valid only for the gases
which don’t react at ordinary conditions  (B)

Example 8 For two gases A and B with molecular weights MA and MB, it is observed that at a certain
temperature T1 the mean velocity of A is equal to the root mean square velocity of B. thus the
mean velocity of A can be made equal to the mean velocity of B if
(A) A is at temperature T and B at T¢, T > T¢
(B) A is lowered to a temperature T2 , T2 < T while B is at T
(C) Both A and B are raised to a higher temperature
(D) Both A and B are placed at lower temperature

8RT 3RT 8 M
Solution: (UAV)A = and (Urms)B =  = A
pMA MB 3p MB

8RT2 8 RT T2 MA 8
for A (UAV) = for B (UAV) = = =  T2 = 8 T or
pMA pM B T MB 3p 3p

T2 < T  (B)

Example 9 The K.E. of N molecule of O2 is x Joules at –123°C. Another sample of O2 at 27°C has a KE
of 2x Joules. The latter sample contains.
(A) N molecules of O2 (B) 2N molecules of O2
(C) N/2 molecules of O2 (D) N/4 molecule of O2
3 3
Solution: Total KE = nRT ; T = – 123 + 273 = + 150 K ;  nR  150 = xJ
2 2
=> 225  8.314  n = x
At 27°C = 27+ 273 = 300K
3
Total KE = 2x Joule =  n1  8.314  300  n1 = n  (A)
2

CENTERS: MUMBAI / DELHI / AKOLA / KOLKATA / LUCKNOW / NASHIK / GOA #


GASEOUS ST AT E Rg - 2015 - 2017
Example 10 If for two gases of molecular weights MA and MB at temperature TA and TB,
TAMB = TBMA, then which property has the same magnitude for both the gases.
(A) density (B) pressure (C) KE per mol (D) Vrms
PM
Solution: i) density of a gas () =
RT
MB M
Since = A ,  at the same pressure  A = B . But if pressure is different
TB TA
then  A  B .
ii) Pressure of the gases would be equal if their densities are equal otherwise not.
3
iii) KE per mol = RT
2
It will be different for the two gases.
3RT TA T
iv) Vrms = , since = B ; Vrms of A = Vrms of B  (D)
M MA MB
Example 11 Helium atom is two times heavier than a hydrogen molecule. At 298 K, the average kinetic
energy of a Helium atom is
(A) two times that of hydrogen molecule (B) same as that of a hydrogen molecule
(C) four times that of a hydrogen molecule (D) half that of a hydrogen molecule
3
Solution: The average kinetic energy of an atom is given as kT.
2
 It does not depend on mass of the atom.  (B)

Example 12 The ratio between the rms velocity of H2 at 50 K and that of O2 at 800 K is
(A) 4 (B) 2 (C) 1 (D) 1/4
3R  50 3R �800
Solution: Vrms (H2 at 50 K) = ; Vrms (O2 at 800K) =
2  10 - 3 32 �10-3
3R  50
Vrms (H2 ) 2  10 - 3 = 25  10 3
= =1 (C)
Vrms (O2 ) 3R  800 25  10 3
32  10 - 3

Example 13 The temperature of an ideal gas is increased from 140 K to 560 K. If at 140 K the root mean
square velocity of the gas molecule is V, at 560 K it becomes
(A) 5V (B) 2V (C) V/2 (D) V/4

Solution: The Vrms at 140K is V

V= 3R  140
M
3R  560 3R  140  4 3R  140
At 540 K, V¢ = = =2 = 2V
M M M

CENTERS: MUMBAI / DELHI / AKOLA / KOLKATA / LUCKNOW / NASHIK / GOA #


GASEOUS ST AT E Rg - 2015 - 2017
 (B)

Example 14 At 100oC and 1 atm, if the density of liquid water is 1.0 g/cc and that of water vapour is
0.0006 g/cc, then the volume occupied by water molecule in one litre of steam at that
temperature is
(A) 6 cc (B) 60 cc (C) 0.6 cc (D) 0.06 cc

Solution: Mass of 1 lt water vapour = V ×d = 1000 × 0.0006 = 0.6g


0 .6
volume of liquid water = = 0.6cc  (C)
1

Example 15 A gas can be liquefied by pressure alone when its temperature is


(A) higher than its critical temperature (B) lower than its critical temperature
(C) either of these (D) none
Solution: A gas can be liquefied only if its temperature is lower than its critical temperature
 (B)

Example 16 The behavior of a real gas is usually depicted by plotting compressibility factor Z versus P at
a constant temperature. At high temperature and high pressure, Z is usually more than one.
This fact can be explained by van der Waals equation when
(A) the constant ‘a’ is negligible and not ‘b’
(B) the constant ‘b’ is negligible and not ‘a’
(C) both constants ‘a’ & ‘b’ are negligible
(D)both the constants ‘a’&‘b’ are not negligible.
 n 2a 
Solution:  P + 2  (V – nb) = nRT
 V 

At high pressures, ‘b’ cannot be ignored as the volume of the gas is very low. At high
temperatures ‘a’ can be ignored
P (V–b) = RT ; PV - Pb = RT

PV = RT + Pb ; PV Pb  (A)
= Z = 1+
RT RT

Example 17 The compressibility factor for a given gas is 0.927 at 273 K and 100 atm.
Calculate the amount of gas required to fill a gas cylinder of 100 L capacity
under given conditions. (Molecular wt of gas is 30
(A) 16.4 Kg (B) 14.44 Kg (C) 4 Kg (D) 10.5 Kg
Solution Since for real gas PV= Z nRT
=> 100 × 100 = 0.927 ×w/30×0.0821×273
=> W = 14.439 Kg  (B)

CENTERS: MUMBAI / DELHI / AKOLA / KOLKATA / LUCKNOW / NASHIK / GOA #


GASEOUS ST AT E Rg - 2015 - 2017
Example 18 Using van der waal’s equation, calculate the constant, ‘a’ (in atm Ltr2 mole-2) when
two moles of a gas confined in a four litre flask exerts a pressure of 11 atm at a
temperature of 300 K. The value of ‘b’ is 0.05 Litre mole-1
(A) 6.5 (B) 2.23 (C) 23.2 (D) .85
Solution Vander waal’s gas equation is
( P + n2a/V2) (V-nb) = nRT
Since V =4 litre, P = 11 atm, T = 300K, b = 0.05 litre mol-1, n = 2
Thus (11+22a/42) (4-2 ×0.05) = 2× 0.082× 300
=> a = 6.5 atm litre2 mol-2  (A)

5.2 SOLVED PROBLEMS (SUBJECTIVE)

Example 1 A compound of P and F was analyzed as follows:


Heating 0.2324 g of compound in a 378 mL container turned all of it into gas, which had a
pressure of 97.3 mm of Hg at 77oC. Then the gas was mixed with calcium chloride solution
which turned all the F to 0.2631 g CaF2. Determine the molecular formula of compound.
Solution Number of moles of gas
pV 97.3  0.378
= = = 1.684  10 -3
RT 760  0.0821  350
0.2324
 Molar mass of gas = = 137.99
1.684  10 - 3
38
Also weight of F in 0.2631 gCaF2 =  0.2631 = 0.128 g
78
 Weight of P in 0.2324 g of gas = 0.2324 – 0.128 = 0.10448 g
Empirical formula of gas P F
Weight 0.1044 0.128
0.1044 0.128
Moles
31 19
0.128 31
Simple ratio 1  = 2
19 0.1044
Empirical formula is PF2 and empirical formula weight = 69
137.99
No. of empirical formula unit in a molecule = =2
69
Hence, molecular formula = P2F4
Example 2 A flask containing some He gas at 1.5 atm and 300 K is connected to another flask, four times
larger in volume containing nitrogen gas at same temperature and pressure by means of a
narrow tube of negligible volume. The bigger flask was then kept in a thermostat bath
maintained at 500 K, while other flask was maintained at constant 300 K. Determine the final
pressures.

CENTERS: MUMBAI / DELHI / AKOLA / KOLKATA / LUCKNOW / NASHIK / GOA #


GASEOUS ST AT E Rg - 2015 - 2017
Solution: Since, initially gases are at same temperature and pressure, if there is x mol of He gas, mole
of N2(g) = 4x (Avogadro’s law)
V 4V
He N2

Final conditions:
n1, 300 K, p n2, 500 K, p
Applying ideal gas laws
 V   V  � V � � 4V �

nT
 =  � � �= � �
 I  nT  II �n1 �300 �
I �n2 �500 �
II

12n1 17n1 25 x
Also, n1 + n2 = x + 4x = 5x = n1 + = � n1 =
5 5 17
Now, applying ideal gas law on flask I, before and after heating.
1.5 x x 17 1.5  25
p �n � = = �17 = p = = 2.2
p n1 25 x 25 17
Thus p =2.2 atm

Example 3 Two flasks of equal volume, connected by a narrow tube of negligible volume contain 1.0
mol of H2 gas at 300 K and 0.5 atmosphere. Now one of the flasks is immersed into a
thermostat maintained at 400 K and other was maintained at constant 300 K temperature.
Determine final pressure and amount of H2 gas in each flask.
Solution: Initially, both the flasks contained Initial conditions: 0.5 mol. 0.5 atm 0.5 mol. 0.5 atm
300 K 300 K
equal moles of H2(g) i.e., 0.5 mol each
(Avogadro’s law). A B

Final conditions: x mol, p atm y mol, p atm


On heating one of the flasks, keeping 300 K 400 K
other at constant temperature, kinetic
energy of the gas molecule present in the hotter flask will increase and they will migrate
towards colder flask. Equilibrium will be re-established when the two flasks acquire a
constant uniform pressure and at equilibrium, amount of gas will be more in colder flask than
in hotter flask
On applying gas equations in final conditions:
300 x = 400 y  3x = 4y
3 7 4 3
Also, x + y = 1  x + x = x =1 x = and y =
4 4 7 7
Now apply gas law on flask A in its initial and final conditions:
0.5 0.5
= p = 4/7 atm
p x

Example 4 A 10.00 cm long column of air is trapped by a column of


mercury, 8.00 cm long in a capillary tube of uniform bore Air Hg

when the tube is held horizontally in a room at 1.0 atm 10 cm 8 cm


pressure as shown below
CENTERS: MUMBAI / DELHI / AKOLA / KOLKATA / LUCKNOW / NASHIK / GOA #
GASEOUS ST AT E Rg - 2015 - 2017
What will be the length of air column when the tube is held
(a) vertically with the open end up?
(b) with the open end down?
(c) at an angle of 45o from vertical with the open end up?
Solution: The laboratory pressure is 1.0 atm = 760 mm of Hg.
If the cross-sectional area of the bore of tube is A, then, the volume of trapped air
of length l will be A  l. p1V1 = p2V2
� p1l1A = p 2 l2 A � p1l1 = p 2l2
The three different situations are described diagrammatically in the following
figures: 8 cm
Hg

Y cm

Air
x cm
Air

8 cm

Hg

(a) pair = patm + pHg (b) pair = patm- pHg


Hg
Air

α = 45o

(c) pair = patm+ pHg


In case of (a)
Pressure on air column = 760 + 80 = 840 mm
p1l1 = p2l2 and, 760  10 = 8 40  x
=> x = 9.047 cm

Hence, length of air column in (a) is 9.047 cm


In case (b)
P(air) = 760 - 80 = 680 mm  Y = 760  10/680 = 11.18 cm
In case (c).
Effective height h of Hg column = 8 sin 45 = 5.657 cm
P(air) = 760 + 56.57 = 816.57 mm  Z= 760  10/816.57 = 9.3 cm

Example 5 A vertical hollow cylinder of height 1.52m is fitted with a movable piston of negligible mass
and thickness. The lower half of the cylinder contains an ideal gas and the upper half is filled
with mercury. The cylinder is initially at 300 K. When the temperature is raised half of the

CENTERS: MUMBAI / DELHI / AKOLA / KOLKATA / LUCKNOW / NASHIK / GOA #


GASEOUS ST AT E Rg - 2015 - 2017
mercury comes out of the cylinder. Find the temperature assuming the thermal expansion of
mercury to be negligible?

Solution: Initially at lower end


P = 76 cm due to Hg + 76 cm due to atm
V
= 152 cm, T = 300K, V = 1
2
(where V1 is the volume of cylinder)
Finally at lower end
P = 76 cm of air + 38 cm of Hg
3 V1
= 114 cm, T = ?, V = 
4
P1V1 P2 V2
=
T1 T2
152  V1 114  3 V1
=  T = 337.5 K
2  300 4T

Example 6 A spherical glass bulb of radius 1.0 m contains a concentric rubber balloon that contains some
N2 gas and the remaining space in the flask contains 50 g H 2. In the given conditions, radius
of the rubber balloon was found to be 40 cm. Now the seal of glass bulb was opened and 25 g
H2(g) was further added and resealed. Determine radius of the rubber balloon in the new
conditions. Assume constant temperature throughout.
Solution:
50 g H2(g) 75 g H2(g)
p p
100- rcm
60 cm

40 cm +25 g H2(g) r cm
N2 N2
p p

I II
From the above illustration, it is obvious that pressure inside and outside the balloon is same
in both the cases. Applying the gas law on I to N2(g) and H2(g); p and T are same:

p (1003 - 403 )
V( H 2 ) V( N 2 ) 4 4
p  403
=  3 = 3
n( H 2 ) n( N 2 ) 25 n( N 2 )

25  (40)3
 n( N 2 ) = = 1.7 mol
[(100) 3 - (40) 3 ]
Applying the gas law on II to N2(g):
4 3
pr 3
37.5 (100) 3 - r 3  100 
4 [(100) - r ]
3 3
 = =  -1
p = 3 1.7 r3  r 
3 37.5 1.7
3
 100  37.5 392
   = +1=  r = 35.13 cm
 r  1.7 17

CENTERS: MUMBAI / DELHI / AKOLA / KOLKATA / LUCKNOW / NASHIK / GOA #


GASEOUS ST AT E Rg - 2015 - 2017
Example 7 A 100 mL flask containing oxygen gas at 1.2 atm and 300 K is connected to a 250 mL flask
containing NO(g) at 0.6 atm and 300 K, by means of a narrow tube of negligible volume,
where they combine quantitatively to form NO2. Finally NO2(g) dimerized partially into N2O4
and pressure inside the flask was found to be 0.41 atm at the same temperature. Determine
the percentage dimerization of NO2 as
2NO2(g)  N2O4(g)
Solution: Before reaction Partial pressure of O2 in the combined system
100
= 1.2  = 0.3428 atm
350
Partial pressure of NO in the combined system
250
= 0.6  = 0.4285 atm
350
250 mL (T)
100 mL (T)

O2(g)
NO(g)
1.2 atm 0.6 atm

Also, in the combined system P1  n1; partial pressure can directly be used to
solve the stoichiometry of reaction as
2NO + O2 l 2NO2 � N2O4
Initial partial p: 0.4285 0.3428 0 0

Partial pressure at the 0 0.12855 0.4285 0


end of reaction I
p
Partial pressure after reaction II: 0 0.12855 0.4285 - p
2
Total pressure p at the end of reaction II,
p
p = 0.55705 - = 0.41
2
 p = 0.2941 atm
0.2941
 % dimerization =  100 = 68.63
0.4285
Thus % dimerization is 68.63 %

Example 8 A 50.00 ft long classroom has twenty


18.8 ft
rows of seats and the adjacent rows
are separated by 2.5 ft each. A N 2O 1.3 ft
CH 4
cylinder on the front door and a CH 4 ��� � N2O
cylinder on the rear door were 1.2 ft
opened simultaneously, so that gases
start diffusing into the
8th row from edge
classroom. Student of which row will 7th row
from front
first experience both tears and from front
laughing? Assume both the cylinders have gases at same temperature and
CENTERS: MUMBAI / DELHI / AKOLA / KOLKATA / LUCKNOW / NASHIK / GOA #
GASEOUS ST AT E Rg - 2015 - 2017
pressure and valves have similar dimensions.
Solution: Since, CH4 (16) is lighter than laughing gas N2O (44), former will diffuse at faster rate. Let
at x ft from the front door, two types of gases meet first. Then according to Graham’s law:
50 - x 44
= = x = 18.8 ft
x 16
The above distance is lying between 7th and 8th row from the front as shown:
Since, CH4 is moving at faster rate than N2O, the former will cover distance of 1.3 ft
(of 7th row) earlier than the later will cover a distance of 1.2 ft (of 8 th row). Hence, the students
seating in 7th row will first experience smell of both N 2O and CH4 simultaneously and they will
tear and laugh together first.

Example 9 Rate of effusion of a sample of ozonized oxygen is 0.95 times the rate of effusion of pure
oxygen. Determine mass percentage of ozone in the ozonized sample.
���
Solution: For ozonisation: 31 O
-a
2 ��� 2O3
2/3a

a
Total mole at equilibrium = 1 -
3
32
� M mix = r (ozonised mix) 32 � a �
a � = 0.95 = 1- �
��
1- r (oxygen) 32 � 3 �
3
 a = 0.2925
Weight of ozone in the mixture = Weight of oxygen ozonised
= 32  0.2925 g.
0.2975  32
 Weight percentage of O3 in mixture  100 = 29.75 %
32
Example 10 Root mean square speed of N2(g) in air is 515 ms-1. What is the average speed of He gas in
the same air?
3RT 8 RT
Solution: urms = and u avg =
M N2 pM He

 Both are at same temperature:


uHe 8M N 2 8 �28 �10-3
� = = = 2.4375
urms ( N 2 ) 3p M He 3p �4 �10-3

 u He = 2.4375  515 = 1255.3ms-1


Example 11 Density of a sample of air was found to be 1.17 g/L at 1.0 atm. Determine average speed of
gas molecules present in air in the given conditions.
8RT 8P 8 101325 Nm -2
Solution: u = and PM = RT ; u = = = 469.6ms-1
pM p p  1.17 kg m - 3

Example 12 Using van der Waals’ equation of state, find the minimum in pV-p curve for one
mole of O2(g) at 0oC.

CENTERS: MUMBAI / DELHI / AKOLA / KOLKATA / LUCKNOW / NASHIK / GOA #


GASEOUS ST AT E Rg - 2015 - 2017
Given: a = 1.362 L2 atm mol-2, b = 0.032 L mol-1.
Solution: The minimum in curve will occur at
 ( pV )
= 0
p
RT a
 p= - 2 … (i)
Vm - b Vm
 ( pVm ) Vm p
Also, = Vm + p = Vm + …(ii)
p p P / Vm
p - RT 2a
From (i) = + 3
Vm (Vm - b) 2
Vm
RT a
- 2
( pVm ) (V - b) Vm
 = Vm + m = 0 at minima
p - RT 2a
+
(Vm - b) 2 Vm3
 (bRT - a) Vm2 + 2abVm - ab 2 = 0 …(iii)
Substituting values of ‘a’ and ‘b’ gives
- 0.647 Vm2 + 0.0869Vm - 0.001386 = 0
 Vm = 0.0185 and 0.1158
The first solution yields negative p and discarded, while second solution yields, p = 166 atm.

 B 
Example 13: One way of writing the equation of state for a real gas is PV = RT 1 + + - - - -  Where B
 V 
is a constant. Derive an approximate expression for B in terms of Vander waal’s constant ‘a’
and ‘b’

 a  RT a
Solution: P + 2  V - b = RT or P= - 2
 V  ( V - b) V
RTV aV  V a 
Multiply by [V] then, PV = - or PV = RT  - 
V -b V2  ( V - b ) VRT 
 -1
a 
or PV = RT 1 -
b
 - 
 V VRT 

-1 2 3  2

Now 1 - b  = 1 + b +  b  +  b  +   PV =RT 1 +
b
-
a b
+   
 V V V  V   V VRT  V  
 

  a  1 b
2
 a
Or PV = RT  
1 + b -   +   +  B=b –
  RT  V  V   RT

Example 14: 10 ml of a gaseous hydrocarbon were burnt completely in 80 ml of O2 at N.T.P. The


remaining gas occupied 70 ml at N.T.P. This volume becomes 50 ml on treatment with
KOH solution. What is the empirical formula of hydrocarbon?
Solution: Let the molecular formula of hydrocarbon be CxHy.

CENTERS: MUMBAI / DELHI / AKOLA / KOLKATA / LUCKNOW / NASHIK / GOA #


GASEOUS ST AT E Rg - 2015 - 2017
CxHy + (x + y/4)O2  xCO2 + y/2 H2O.
Volume reacted 10 ml 10(x + y/4) - -
Volume produced 10x negligible
� 10x = 20 � x = 2.
Since, the remaining gas occupied 70 ml,
 Total volume left = 70 = VO 2 + VCO 2
where VO 2 = volume of oxygen left and VCO 2 = Volume of CO2 produced.
 VO 2 = 70 - 20 = 50 ml.
80 - 10(x + y/4)= 50
x + y/4 = 3
y/4 =1 y = 4.
So, formula is C2H4.
Thus, empirical formula of the hydrocarbon is CH2.
Example : 15 A mixture of ethane (C2H6) and ethene (C2H4) occupies 40 litre at 1.00 atm and at 400 K. The
mixture reacts completely with 130 g of O2 to produce CO2 and H2O. Assuming ideal gas
behaviour, calculate the mole fractions of C2H4 and C2H6 in the mixture.
1  40
Solution: Number of moles of C2H6 and C2H4 = = 1.218
0.0821  400
Let the number of moles of C2H6 in the mixture be x
7
C2H6 + O2  2CO2 + 3H2O
2
7x
x
2
C2H4 + 3O2  2CO2 + 2H2O
1.218-x 3(1.218-x)
7x 130
Number of moles of O 2 = + 3 (1.218 - x ) =
2 32
On solving, x = 0.817
0.817
Moles fraction of C2H6 = = 0.67
1.218
Mole fraction of C2H6 = 0.33

CENTERS: MUMBAI / DELHI / AKOLA / KOLKATA / LUCKNOW / NASHIK / GOA #


GASEOUS ST AT E Rg - 2015 - 2017

6.1 FOUNDATION BUILDERS (OBJECTIVE)

Experimental Gas Laws

1. I, II, III are three iso-therm respectively at T1, T2 & T3 temperatures will be in order
(A) T1 = T2 = T3 (B) T1 < T2 < T3 P

(C) T1  T2  T3 (D) T1  T2 = T3 II
I

2. The volume of a large irregularly shaped tank is determined as follows. The tank is V
III

first evacuated, and then it is connected to a 50 L cylinder of compressed helium gas.


The gas pressure in the cylinder, originally at 21atm, falls to 7.0atm without a change in temperature.
What is the volume of the tank?
(A) 100L (B) 150L (C) 200L (D) 300L

3. At a constant temperature what should be the percentage increase in pressure for a 5% decrease in
the volume of gas:
(A) 5% (B) 10% (C) 5.26% (D) 4.26%

4. At a constant pressure, what should be the percentage increase in the temperature in Kelvin for a
10% increase in volume
(A) 10% (B) 20% (C) 5% (D) 50%

5. According to Charles law


(A)  dV  = K (B)  dV  = -K (C)  dV  = - K (D) none
 dT  p  dT  P  dT  p T

6. The volume of gas is 100ml at 100 oC. If pressure remains same then at what temperature it will be
200ml?
(A) 200oC (B) 473oC (C) 746oC (D) 50oC

7. A gas cylinder containing cooking gas can withstand a pressure of 14.9 atm. The pressure gauge of
cylinder indicates 12 atm at 27°C. Due to sudden fire in building the temperature starts rising. The
temperature at which the cylinder will explode is.
(A) 42.5°C (B) 67.8°C (C) 99.5°C (D) 25.7°C

Ideal Gas Law

8. Flasks A and B of equal size contain 2 gm of H2 and 2 gm of N2 respectively at the same temperature.
The number of molecules in flask A is
(A) Same as those in flask B (B) Less than those in flask B
(C) Greater than those in flask B (D) Exactly half of those in flask B

9. 3.2g S on heating if occupy a volume of 780 ml at 450°C and 723 mm pressure. Formula of sulphur
is:
(A) S2 (B) S (C) S4 (D) S8
10. The volume of balloon filled with 4.0g He at 22oC and 720 mm of Hg is
(A)25.565 L (B) 20 L (C) 15 L (D) 30 L

CENTERS: MUMBAI / DELHI / AKOLA / KOLKATA / LUCKNOW / NASHIK / GOA #


GASEOUS ST AT E Rg - 2015 - 2017
11. Two containers A and B contain the same gas. If the pressure, volume and absolute temperature of
the gas in A are twice as compared to that of gas in B, and if the mass of the gas B is x g, the mass of
gas in A is
(A) x g (B) 4x g (C) 2/x g (D) 2x g

12. What is the final temperature if a sample of ammonia gas, initially at a pressure of 3.00 atm, a
temperature of 500K, and a volume of 275L is changed to a volume of 200L and a pressure of
2.50atm?
(A) 303K (B) 436K (C) 573K (D) 825K

13. 5.40 gm of an unknown gas at 27C occupies the same volume as 0.14 gm of hydrogen at 17C and
same pressure. The molecular weight of unknown gas is
(A) 79.8 (B) 81 (C) 79.2 (D) 83

14. Which of the following gases would have the largest density at 25� C and 1.00 atm pressure?
(A) Methane, CH4 (B) Acetylene, C2H2 (C) Ethylene, C2H4 (D) Propane, C3H8
15. To expel half the mass of air from a large flask at 27oC, it must be heated to:
(A) 54oC (B) 177oC (C) 277oC (D) 327oC

16. An open vessel containing air is heated from 300 K to 400 K. The fraction of air originally present
which goes out of it is:
(A) 3/4 (B) 1/4 (C) 2/3 (D) 1/8

17. 0.2 mole sample of hydrocarbon CxHy yields after complete combustion with excess O2 gas, 0.8 mole
of CO2, 1.0 mole of H2O. Hence hydrocarbon is
(A) C4H10 (B) C4H8 (C) C4H5 (D) C8H16
18. If the pressure of a given mass of gas is reduced to half and temperature is doubled simultaneously
then the volume will be
(A) Same as before (B) Twice as before (C) ¼ the as before (D) None

19. The 1 mol of an Ideal gas A with 300 mm of Hg is separated by 2 mol of another ideal gas B with
300 mm of Hg in closed container at the same temperature. If the separation is removed than total
pressure is
(A) 200 mm of Hg (B) 300 mm of Hg (C) 500 mm of Hg (D) 600 mm of Hg

20. Assume centre of sun to consist of gases whose average molecular weight is 2. The density and
pressure of the gas are 1.3 g cc–1 and 1.12  109 atm respectively. The temperature of sun is
(A) 2  103 K (B) 2  105 K (C) 2  107 K (D) 2  109 K

21. 6 g each of the following gases at 87oC and 750 mm pressure are taken. Which of them will have the
least volume
(A) HF (B) HCl (C) HBr (D) HI

22. The density of O2 gas at 25oC is 1.458 mg/lt at one atm pressure. At what pressure will O 2 have the
density twice the value?
(A) 0.5 atm/250C (B) 2atm/25oC (C) 4atm/25oC (D) none

23. The density of neon gas is highest at


(A)STP (B) 0oC, 2 atm (C) 273oC, 1 atm (D) 273oC, 2 atm

CENTERS: MUMBAI / DELHI / AKOLA / KOLKATA / LUCKNOW / NASHIK / GOA #


GASEOUS ST AT E Rg - 2015 - 2017
24. Figure shows graphs of pressure versus density for an ideal gas at two temperatures T1 and T2.
Which is correct?

Pressure
Density

(A) T1 > T2 (B) T1 = T2 (C) T1 < T2 (D) none of these

25. Equal masses of three ideal gases X, Y and Z are mixed in a sealed rigid container. If the
temperature of the system remains constant, which of the following statements about the partial
pressure of gas X is correct?
(A) It is equal to 1/3 the total pressure
(B) It depends on the intermolecular forces of attraction between molecule of X, Y and Z.
(C) It depends on the relative molecular masses of X, Y and Z.
(D) It depends on the average distance travelled between molecular collisions.

Dalton’s Law Of Partial Pressure

26. When 2g gas A is introduced into an evacuated flask kept at 25°C, the pressure is found to be 1 atm.
If 3g of another gas B is further added to same flask, the total pressure becomes 1.5 atm. The ratio of
molecular weights is
(A) 1:1 (B) 1:2 (C) 1:3 (D) 1:4
27. Air contains 79 % N2 and 21 % O2 by volume. If the barometric pressure is 750 mm Hg. The partial
pressure of oxygen is
(A) 157.5 mmHg (B) 175.5 mmHg (C) 315.0 mmHg (D) none

28. A gaseous mixture contains 1g of H2, 4g of He, 7g of N2 and 8g of O2. The gas having the highest
partial pressure is
(A) H2 (B) O2 (C) He (D) N2
29. A mixture consisting of 0.10 moles of N2, 0.05 moles of O2 and 0.20 moles of CH4 and an unknown
amount of CO2 occupied a volume of 9.6 L at 27� C and 1.0 atm pressure. How many moles of CO 2
are there in this sample ?
(A) 0.04 mol (B) 0.39 mol (C) 0.05 mol (D) 0.10 mol

Graham’s Law

2
30. X ml of H gas effuses through a hole in a container in 5 secs. The time taken for effusion of same

volume of gas specified below under identical condition is


(A) 10 secs : He (B) 20 secs : O2 (C) 25 secs : CO (D) 55 secs : CO2

31. Which of the following pair will diffuse at the same rate?
(A) CO2 and N2O (B) CO2 and NO (C) CO2 and CO (D) N2O and NO

CENTERS: MUMBAI / DELHI / AKOLA / KOLKATA / LUCKNOW / NASHIK / GOA #


GASEOUS ST AT E Rg - 2015 - 2017
32. A mixture of H2 and O2 in 2:1 volume is allowed to diffuse through a porous partition what is the
composition of gas coming out initially
(A) 1:2 (B) 4:1 (C) 8:1 (D) 1:4

33. The valves X and Y in the adjoining figure are


NH3 HCl
opened simultaneously. The white fumes of NH4Cl
will first formed at B A C
X Y
(A) A (B) B
(C) C (D) A, B and C simultaneously

34. The pair of gases which can be most easily separated from effusion technique
(A) D2 and H2 (B) CH4 and CD4 (C) C12H4 and C14H4 (D) U235F6 and U238F6
35. The rate of diffusion of methane at a given temperature is twice that of gas X. The molecular weight
of X is:
(A) 64.0 (B) 32.0 (C) 4.0 (D) 8.0

36. Which of the following mixture of gases cannot be separated by diffusion method
(A) NO + C2 H6 (B) NO + NO2
(C) CO + CO2 (D) C2H4 + C2 H6

37. Bromine vapour at a given temperature is roughly 5 times denser than oxygen gas. Calculate the
relative rates at which Br2(g) and O2(g) diffuse.
(A) O2 should diffuse roughly 2.2 times faster.
(B) Br2 should diffuse roughly 2.2times faster.
(C) O2 should diffuse roughly 5.0 times faster.
(D) Br2 should diffuse roughly 5 times faster.

38. A balloon filled with moist air has developed a pinhole. It is quickly plunged into a tank of dry air at
the same pressure. In a short while
(A) It will collapse (B) It will enlarge
(C) No change will take place (D) can’t be predicted

39. Some moles of SO2 diffuse through a small opening in 20 seconds. Same number of moles of an
unknown gas diffuses through the same opening in 60 seconds. Molecular mass of the unknown gas
is
�60 � �20 �
(A) ( 64 ) X � � (B) ( 64 ) X � �
2 2

�20 � �60 �
2 2
�60 � �20 �
(C) ( 64 ) X � � (D) ( 64 ) X � �
�20 � �60 �

40. Vegetables are canned, while they are steaming hot because
(A) the heat inside will seal the jars
(B) the heat increases the atmospheric pressure
(C) the heat creates more pressure inside the jars
(D) when the jars cool, a vacuum inside will help to seal the jars
Eudiometry

CENTERS: MUMBAI / DELHI / AKOLA / KOLKATA / LUCKNOW / NASHIK / GOA #


GASEOUS ST AT E Rg - 2015 - 2017

41. 200 ml of a gaseous mixture containing CO, CO 2 and N2 on complete combustion in just sufficient
amount of O2 showed contraction of 40 ml when the resulting gases were passed through KOH
solution it reduces by 50% then calculate the volume ratio of VCO2 : VCO : VN2 in original mixture.
(A) 4: 1: 5 (B) 2: 3: 5 (C) 1 : 4 : 5 (D) 1 : 3: 5

42. 10 ml of a compound containing ‘N’ and ‘O’ is mixed with 30 ml of H 2 to produce H2O (l) and 10 ml
of N2 (g). Molecular formula of compound if both reactants reacts completely, is
(A) N2O (B) NO2 (C) N2O3 (D) N2O5

43. One mole mixture of CH4 & air (containing 80% N2 20% O2 by volume) of a composition such that
when underwent combustion gave maximum heat (assume combustion of only CH 4). Then which of
the statements are correct, regarding composition of initial mixture.(X presents mole fraction)
1 2 8 3 1 1
X CH 4 = , X O2 = , X N 2 = X CH 4 = , X O2 = , X N 2 =
(A) 11 11 11 (B) 8 8 2
1 1 2
X CH 4 = , X O2 = , X N2 =
(C) 6 6 3 (D) Data insufficient

44. A mixture of C2H2 and C3H8 occupied a certain volume at 80 mm Hg. The mixture was completely
burnt to CO2 and H2O(l). When the pressure of CO2 was found to be 230 mm Hg at the same
temperature and volume, the fraction of C2H2 in mixture is
(A) 0.125 (B) 0.5 (C) 0.85 (D) 0.25

Kinetic Theory of Gases

45. If Ek is the average kinetic energy per mole of a gas, then


3 3
(A) PV = Ek (B) P = VEk
2 2
2
(C) PV = Ek (D) 3 PV = Ek
3
46. The kinetic energy of a mole of ideal gas in calories is approximately equal to
(A) 3 times its absolute temperature
(B) 2 times its absolute temperature
(C) 4 times its absolute temperature
(D) 2/3 times its absolute temperature

47. A sample of gas is at 0oC. The temperature at which rms speed of the molecule will be doubled is
(A) 103oC (B) 273oC (C) 723oC (D) 819oC

48. The temperature at which H2 has same rms speed (at 1 atm) as that of O2 at NTP is
(A) 37 K (B) 17 K (C) 512 K (D) 27 K

49. In a closed vessel, a gas is heated from 300 K to 600 K the kinetic energy becomes/remain
(A) half (B) double (C) same (D) four times

50. Which of the following gases would have the highest rms speed at 0 oC
(A) O2 (B) CO2 (C) SO3 (D) CO

CENTERS: MUMBAI / DELHI / AKOLA / KOLKATA / LUCKNOW / NASHIK / GOA #


GASEOUS ST AT E Rg - 2015 - 2017
51. The ratio of rms velocity to average velocity of gas molecules at a particular temperature is
(A) 1.086 : 1 (B) 1 : 1.086 (C) 2 : 1.086 (D) 1.086 : 2

52. On increasing temperature, the fraction of total gas molecule which has acquired most probable
velocity will
(A) Increase (B) decrease
(C) Remains constant (D) cant say without knowing pressure

53. Distribution of molecules with velocity is represented by the


curve. Point A in the curve shifts to the higher value of velocity if
(A) T is increased
(B) V is increased
(C) P is increased
(D) All
54. The K.E. of N molecule of O2 is x Joules at –123°C. Another sample of O2 at 27°C has a KE of 2x
Joules. The latter sample contains.
(A) N molecules of O2 (B) 2N molecules of O2
(C) N/2 molecules of O2 (D) N/4 molecule of O2
55. The ratio between the rms velocity of H2 at 50 K and that of O2 at 800 K is
(A) 4 (B) 2 (C) 1 (D) 1/4

56. Let the most probable velocity of hydrogen molecules at a temp tC is Vo. Suppose all the molecules
dissociate into atoms when temp is raised to (2 t + 273)C then the new r.m.s velocity is
(A) 2 / 3 V0 (B) 3(2 + 273 / t ) V0
(C) 2 3 V0 (D) 6 V0
57. An ideal gas molecule is present at 27C. By how many degree centigrade its temperature should be
raised so that its Vrms , Vmp and Vav all may double.
(A) 900C (B) 108C (C) 927C (D) 81C

58. If most probable speed is represented by mps and fraction of molecules possessing this speed by f,
then with increase in temperature
(A) both mps and f will increase (B) mps will increase but f will decrease
(C) Both will decrease (D) mps will decrease but f will increase

59. Which of the following statements concerning the kinetic theory of gases is (are) correct?
I. Molecules make elastic collisions with each other and with the walls of their container.
II. The average kinetic energy of a large number of molecules of mass, M, is proportional to M 1/2 at
a given temperature.
III. The molecules of a gas are in constant random motion.
IV. All the molecules of a gas have the same kinetic energy at a given temperature.
(A) I,II,III,IV (B) I,II,III (C) I,III,IV (D) I,III

60. The root mean square speed of CH 4 molecules at 25�C is about 0.56 km/s. What is the root mean
square speed of a H2 molecule at 25�C?
(A) 0.070 km/s (B) 0.20 km/s (C) 1.1 km/s (D) 1.6 km/s

CENTERS: MUMBAI / DELHI / AKOLA / KOLKATA / LUCKNOW / NASHIK / GOA #


GASEOUS ST AT E Rg - 2015 - 2017

61. Assume that the container is filled with the mixture of SO 3 and Ne. The molecular weight of SO3 is
80 g/mol and the atomic weight of Ne is 20 g/mol. The average velocity of an SO3 molecules is
(A) One fourth that of a Ne atom (B) One half that of a Ne atom
(C) The same as a Ne atom (D) Two times that of a Ne atom

62. Consider two 1 L flasks, one containing O2, the other containing He, each at STP. Which of the
following statement is NOT true regarding these gases?
(A) Each flask contains the same number of atom or molecule
(B) The gases in each flask have the same average kinetic energy.
(C) The gases in each flask have the same density.
(D) The pressure in each flask is the same.

63. Compare the root mean square speed of an O 2 molecule with that of CH4 molecule at the same
temperature and pressure.
(A) The speed are the same, since the weight of O2 and of CH4 are both 16 g/mol
(B) The speed are the same because at the same temperature all gas molecules have the same mean
square speed.
(C) CH4 is 2.00 times faster, because the MW of O2 is times greater than the molecular weight of
CH4.
(D) CH4 is 1.41 times faster, since at equal temperature of all gas molecules have the same kinetic
energy. Square root mean square speed then is inversely proportional to the square root of the
molecular weight.

64. The kinetic molecular theory of gases predicts pressure to rise as the temperature of a gas increases
because
(A) The average kinetic energy of the gas molecules decreases
(B) Gas molecules collide more frequently with the container walls
(C) Gas molecules collide less frequently with the container walls
(D) Gas molecules collide less energetically with the container walls

65. “The higher the temperature of a gas at constant volume, the greater is its pressure.” Which of the
following kinetic molecular explanations of this principle is not correct
(A) Molecule move faster at higher temperatures
(B) The number of molecular impacts per unit area decreases at a higher temperature.
(C) The average kinetic energy of molecules is greater at a higher temperature.
(D) The average momentum and rate of molecular collisions is greater at a higher temperature.

66. A certain gas is at a temperature of 350 K. If the temperature is raised to 700K, the average
translational kinetic energy of the gas will
(A) Remains constant
(B) Increase by a factor of 2
(C) Increase by a factor of square root of 2
(D) Decrease by a factor of square root of 2

67. Identify a postulate of Kinetic theory among the following


(A) An atom is indivisible
CENTERS: MUMBAI / DELHI / AKOLA / KOLKATA / LUCKNOW / NASHIK / GOA #
GASEOUS ST AT E Rg - 2015 - 2017
(B) Gases combine in simple ratio
(C)There is no influence of gravity on gas molecules
(D) None of the above

68. Molecular velocities of two gases at the same temperature are U 1 and U2 and their molecular masses
are m1 and m2 respectively. Which of the following expression is correct
m1 m2 m1 m2
(A) = (B) m1 U1 = m2U2 (C) = (D) m1U12 = m2 U22
U12 U 22 U1 U 2

69. If P is the pressure of gas, then the kinetic energy per unit volume of the gas is
(A) P/2 (B) P (C) 3P/2 (D) 2P

70. If the absolute temperature of a sample of gas in a fixed volume container is quadrupled, then the
root mean square speed in the initial state ui and that in the final stage uf would be related as:
(A) uf = ui/4 (B) uf = ui/2 (C) uf = 2ui (D) uf = 4ui
71. Consider the following statement about Maxwell Boltzmann law of distribution of molecular speeds:
1) The fraction of molecules having speed between c and (c + dc) (regardless of direction) is given
3/2
�M � 2 � Mc 2 �
by 4π � � c exp �- �dc
�2p RT � � 2RT �
2) The average speed is the arithmetic mean of the different speeds of all the molecules present in a
given sample of the gas.
3) The speed distribution curve becomes sharper and is more peaked at higher temperature as the
average speed increases.
4) The speed distribution function is used to determine average molecular speeds.
of these statements:
(A) 1, 2 and 3 are correct (B) 1, 2 and 4 are correct
(C) 1,3 and 4 are correct (D) 2, 3 and 4 are correct

Compressibility Factor
72. The compressibility factor for an ideal gas is
(A) 1.5 (B) 1 (C) 2 (D) 

73. The compressibility factor of He as a real gas at room temperature is


a Pb RTV
(A) Unity (B) 1 - RTV (C) 1 + RT (D) 1- a
74. For a non-zero volume of molecules having no force of attraction,
the variation of compressibility factor, Z vs P is best represent by
(A) I (B) II Z II
1.0
(C) III (D) All the above I
III

P (atm)

75. In a given diagram gas X and Y can be


(A) Hydrogen and Helium (B) Helium and Hydrogen
(C) Hydrogen and Neon (D) Helium and Neon

CENTERS: MUMBAI / DELHI / AKOLA / KOLKATA / LUCKNOW / NASHIK / GOA #


GASEOUS ST AT E Rg - 2015 - 2017

76. The plot of Z vs P for N 2 gas at different temperature is shown,


correct order of the temperature T1, T2 and T3 is T3
Z
(A) T1 > T2 > T3 (B) T1 < T2 > T3 1.0

T2
(C) T1 < T2 < T3 (D) T1 = T2 = T3 T1

P (atm)

Van der Waal’s gas equation


77. For non-zero value of force of attraction between gas molecules gas equation will be
n 2a nRT
(A) PV = nRT – (B) PV = nRT + nbP (C) PV = nRT (D) P =
V V-b

78. Out of the following gases, which one has least value of Vander Waals constant ‘a’
(A) CO2 (B) NH3 (C) CH4 (D) H2

79. The value of Vander Waals constant ‘a’ is maximum for


(A) Helium (B) nitrogen (C) CH4 (D) NH3

80. Which of the following statement is true for Van der waals gas constant ‘a’ and ‘b’?
(A) ‘a’ depends on size and shape, ‘b’ depends only on size of molecule.
(B) ‘b’ depends on size and shape, ‘a’ depends only on size of molecule.
(C) Both ‘a’ and ‘b’ depends on shape and size of molecule.
(D) Both ‘a’ and ‘b’ depends only on size of molecule.
81. Table gives values of ‘a’ for different gases
O2 H2 NH3 CH4
1.310 1.390 4.17 2.253
Therefore, the one which can be most easily liquefied is
(A) O2 (B) NH3 (C) H2 (D) CH4
82. Identify the conditions of pressure and temperature at which a real gas shows maximum deviation
from ideal behavior:
(A)10 atm, 273 K (B) 5 atm, 273 K (C) 10 atm, 373 K (D) 5 atm, 373 K

83. a/V2 given in van der Waals equation is for:


(A) internal pressure (B) intermolecular attraction
(C) both 1 and 2 (D) temperature correction.

84. Which of the following statement is not true ?


(A) The pressure of the real gas is equal to the pressure calculated for an ideal gas.
(B) The van der Waals’ equation helps to calculate the pressure and volume of real gases
(C) Real gas molecules do occupy a finite, but small, volume.
(D) None of these

85. The compressibility factor for gas obeying van der Waals’ equation of state is given by ( where V is
molar volume)

CENTERS: MUMBAI / DELHI / AKOLA / KOLKATA / LUCKNOW / NASHIK / GOA #


GASEOUS ST AT E Rg - 2015 - 2017

V a a V V - b RTV RTV V - b
(A) - (B) - (C) - (D) -
V - b RTV RTV V - b V a a V

Virial Gas Equation and Liquefaction of gas

86. One way of writing the equation of state for one mole of real gas
� B C �
PV = RT �
A + + 2 ................�
� V V �
Where A, B and C may be constants or dependent variables.
1. A is temperature dependent, and higher terms B, C………..are temperature independent.
2. A is temperature independent, and higher terms B, C………..are temperature dependent.
3. At Boyle’s temperature, A is unity and B, C….are negligible.
4. At Boyle’s temperature, all terms A, B, C…..are negligible.
Select the correct statements
(A)1, 4 (B) 2, 3 (C) 1, 3 (D) 2, 4

87. The temperature at which a real gas obeys the ideal gas laws at fairly wide range of pressure is
(A) Critical temperature (B) Inversion temperature
(C) Boyle’s temperature (D) Reduced temperature

PV a
88. The compressibility factor Z=
nRT
of a gas above T=
Rb
will be
(A) Always less than Unity (B) Always equal to unity
(C) Always greater than Unity (D) Depends on pressure

89. A given gas cannot be liquefied if its temperature is


(A) equal to its critical temperature (B) greater than its critical temperature
(C) smaller than its critical temperature (D) equal to its inversion temperature

90. Which of the following gases is the least likely to behave ideally?
(A) He (B) N2 (C) HCl (D) H2
91. At moderate pressure, the compressibility factor for a particular gas is given by:
160p
Z = 1 + 0.34p– (p in bar and T in kelvin). What s the Boyle’s temperature of this gas?
T
(A) 298K (B) 340K (C) 470K (D) 680K

92. A gas can be liquefied most suitably at:


(A) T = Tc and P < Pc (B) T < Tc and P = Pc
(C) T < Tc and P > Pc (D) T > Tc and P > Pc
93. NH3 gas is liquefied more easily than N2. Hence:
(A) Van der Waals’ constant a and b of NH3 > than of N2
(B) Van der Waals’ constant a and b of NH3 < that of N2
(C) a (NH3) > a (N2) but b (NH3) < b (N2)
(D) a (NH3) < a (N2) but b (NH3) > b (N2)

6.2 FOUNDATION BUILDERS (SUBJECTIVE)


Experimental Gas Laws and Ideal Gas Law

CENTERS: MUMBAI / DELHI / AKOLA / KOLKATA / LUCKNOW / NASHIK / GOA #


GASEOUS ST AT E Rg - 2015 - 2017
1. An iron cylinder contains helium at a pressure of 250 kPa and 27 0C. The cylinder can withstand a
pressure of 106 Pa. The room in which cylinder is placed catches fire. Predict whether the cylinder
blow up before it melts or not (mp of cylinder = 1800K)

2. a) How large a balloon could you fill with 4.0 g of He gas at 22 oC and 720 mm of Hg?
b) Calculate the density of CO2 at 100 0C and 800 mm Hg pressure.

3. a) A container has 3.2 g of a certain gas at NTP. What would be the mass of the same gas contained
in the same vessel at 200C and 16 atm pressure?
b) A certain quantity of a gas measured 500 mL at a temperature of 15°C and 750 mm Hg. What
pressure is required to compress this quantity of gas into a 400 mL vessel at a temperature of 50°C?

4. An open vessel at 27C is heated until three-fifths of the air in it has been expelled. Assuming the
volume of the vessel remains constant, find the temperature to which the vessel has to be heated.

5. At 60°C the vapour density of N2O4 gas was found to be 30.2. Calculate the percentage of NO 2
molecules by moles.

6. 100 mL of gas is collected at 750 mm pressure. What volume will it occupy at 745 mm pressure?

7. A toy balloon originally held 1.0 gm of He gas and had a radius 10 cm. During the night, 0.25 gm of
the gas effused from the balloon. Assuming ideal gas behaivour, under these constant P and T
conditions, what was the radius of the balloon the next morning?

8. 5 L of nitrogen measured at 750 mm have to be compressed into an iron cylinder of 1 L respectively.


If temperature is kept constant, calculate the pressure in atmospheres required to do so

9. Three hollow metallic boxes A,B and C are connected through a narrow tube of negligible volume
and filled with He gas. If the internal volumes of these boxes are in the ratio of 1:2:4, (i) Find the
molar ratio of He in these boxes (ii) How the molar ratio of He would change if the boxes A,B and
C are immersed in different temperature baths having temperature 27 0C,1270C,&3270C,
respectively?

10. The temperature of a given mass of air was changed from 15 0C to – 150C. If the pressure remains
unchanged and the initial volume was 100 mL, what should be the final volume?

11. The density of a gas at 270C and 760 mm pressure is 24. Calculate the temperature at which it will be
18, the pressure remaining constant.

Dalton’s Law of Partial Pressure

12. 12 g N2, 4 gm H2 and 9 gm O2 are put into a one liter container at 27°C. What is the total pressure?

13. 1.0×10–2 kg of hydrogen and 6.4×10–2 kg of oxygen are contained in a 10×10–3 m3 flask at 473 K.
Calculate the total pressure of the mixture. If a spark ignites the mixture. What will be the final
pressure?

14. Exactly 100 mL of oxygen is collected over water of 23 0C and 800 mm pressure. Calculate the
volume of dry oxygen at NTP. (Vapour pressure of water at 230C is 21 mm.)

CENTERS: MUMBAI / DELHI / AKOLA / KOLKATA / LUCKNOW / NASHIK / GOA #


GASEOUS ST AT E Rg - 2015 - 2017
15. 250 mm of nitrogen gas maintained at 650 mm pressure and 380 mL of oxygen gas maintained at
650 mm pressure are put together in 1 L flask. If temperature is kept constant, what will be the final
pressure of the mixture?

16. At 627oC and 1.0 atm, SO3 partially dissociates into SO2 and O2. One liter of the equilibrium mixture
weighs 0.94 g under the above conditions. Determine the partial pressure of the constituent gases in
the mixture.

Graham’s Law

17. At room temp , NH3 gas at one atm & HCl gas at “P” atm are allowed to effuse through identical
pin holes to the opposite ends of a glass tube 1m long & uniform cross-section. A white deposit is
observed at a distance of 60 cm from the HCl end. What is “P”.

Q.18 The relative of diffusion of ozone as compared to chlorine is 6:5. If the density of Cl 2 is 35.5, find
out the density of ozone.

Q.19 127 mL of a certain gas diffusion of the same time as 100 mL of chlorine under the same conditions.
Calculate the molecular weight of the gas.

20. The ratio of rates of effusion of two gases A and B is 1:4. If the ratio of their masses present in the
mixture is 2:3. What is the ratio of their mole fraction in mixture?

21. A gaseous mixture consists of CH4, O2 and SO2 in the weight ratio of 4:2:1. The mixture was then
allowed to effuse through a small hole at a definite temperature. What will be the mole composition
of the mixture leaving the hole initially?

Q.22 Calculate the molecular weight of a gas X which diffuses four times as fast as another gas Y, which in
turn diffuses twice as fast as another gas Z. Molecular weight of the gas Z is 128.

Eudiometry

23. 10 mL of a gaseous hydrocarbon was burnt completely in 80 mL of O 2 at NTP. The remaining gas
occupied 70 mL at NTP. This volume became 50 mL on treatment with KOH solution. What is the
formula of the hydrocarbon.

24. 7.5 mL of a gaseous hydrocarbon was exploded with 36 mL of oxygen. The volume of gases on
cooling was found to be 28.5 mL, of which 15ml was absorbed by KOH and the rest was absorbed in
a solution of alkaline pyrogallol. If all volumes are measured under the same conditions, deduce the
formula of the hydrocarbon

25. 10 mL of a gaseous organic compound containing C , H and O only was mixed with 100 mL of
oxygen and exploded under conditions which allowed the water formed to condense. The volume of
the gas after explosion was 90 mL. On treatment with potash solution, a further contraction of 20
mL, in volume was observed. Given that the vapour density of the compound is 23, deduce the
molecular formula. All volume measurements were carried out under the same conditions.
26. When 30 mL of a gaseous hydrocarbon is exploded with 200 mL of oxygen, the volume contraction
after explosion was found to be 45 mL. On further treatment with alkaline pyrogallol there was a

CENTERS: MUMBAI / DELHI / AKOLA / KOLKATA / LUCKNOW / NASHIK / GOA #


GASEOUS ST AT E Rg - 2015 - 2017
reduction of 95mL. Find formula of the hydrocarbon. (All volumes are measured at room
temperature).

27. 50 ml of pure and dry oxygen was subjected to slient electric discharge and on cooling to the original
temperature , the volume of ozonised oxygen was found to be 47 mL. The gas was then brought in
contact with turpentine oil, when after the absorption of ozone, the remaining gas occupied a volume
of 41ml. Find the molecular formula of ozone?

28. 10 mL of a mixture of CO , CH 4 and N2 , exploded with excess of oxygen, gave a contraction of 6.5
mL. There was a further contraction of 7 mL when the residual gas treated with KOH. What is the
composition of the original mixture?

29. 7.5 ml of hydrocarbon gas was exploded with excess of oxygen. On cooling it was found to have
undergone a contraction of 15 ml. if vapour density of hydrocarbon is 14. Determine its molecular
formula.

Kinetic Theory Of Gas

30. a) Calculate the rms speed in cm/sec at 25°C of a free electron


b) The flask (A) and (B) have equal volumes. Flask (A) contains H 2 gas at 300K, while (B) contains
equal mass of CH4 at 900K. Calculate the ratio of average speed of molecules in flask (A) and (B)

31. a) Calculate total energy of one mole of an ideal monatomic gas at 27C?
b) How much thermal energy should be added to 3.45 g Ne in a 10 litre flask to raise the temperature
from 0°C to 100°C. (Atomic weight Ne = 20.18)?

32. At what temperature in °C, the Urms of SO2 is equal to the average velocity of O2 at 27°C.

33. Calculate Urms of molecules of H2 at 1 atm, density of H2 is 0.00009 g/cc.

34. The density of CO at 273 K and 1 atm is 1.2504 kg m –3. Calculate (a) root mean square speed(b) the
average speed and (c) most probable speed.

35. At a certain temperature the speeds of six gaseous molecules in a container are 2.0, 2.2, 2.6, 2.7, 3.3,
and 3.5 ms–1. Calculate the root mean square speed and the average speed of the molecules.

36. A bulb of capacity 1 dm3 contains 1.03 × 1023 H2 molecules & pressure exerted by these molecules is
101.325 kPa. Calculate the average square molecular speed and the temperature.

Real Gas Behavior

37. The critical temperature and pressure of CO2 gas are 304.2K and 72.9 atm respectively. What is the
radius of CO2 molecule assuming it to behave as Vander Waal’s gas?

38. Argon has TC= –122°C PC = 48 atm. What is the radius of the argon atm?

39. The density of mercury is 13.6 g/cm3. Estimate the b value.

40. Calculate the pressure exerted by 22 g of carbon dioxide in 0.5 dm3 at 298.15 K using:
(a) the ideal gas law and (b) vander waals equation. Given:
CENTERS: MUMBAI / DELHI / AKOLA / KOLKATA / LUCKNOW / NASHIK / GOA #
GASEOUS ST AT E Rg - 2015 - 2017
[a = 363.76 kPa dm6 mol–2 and b = 42.67 cm3 mol–1]

41. At 273.15 K and under a pressure of 10.1325 MPa, the compressibility factor of O 2 is 0.927.
Calculate the mass of O2 necessary to fill a gas cylinder of 100 dm3 capacity under the given
conditions.

42. The vander waals constant for O2 are a = 1.36 atm L2 mol–2 and b = 0.0318 L mol–1. Calculate the
temperature at which O2 gas behaves, ideally for longer range of pressure.

43. The vander Waals constants for gases A, B and C are as follows
Gas a/dm6 kPa mol–2 b/dm3 mol–1
A 405.3 0.027
B 1215.9 0.030
C 607.95 0.032
Which gas has (i) the highest critical temperature, (ii) the largest molecular volume, and (iii) most
ideal behavior around STP?

44. Calculate the pressure of 15 mol neon at 30oC in a 12 lit container using
(i) the ideal gas equation (ii) the vandar waals equation
[a = 0.2107 atm lit2 mol–2, b = 0.0171 lit mol–1]

45. The density of water vapour at 327.6 atm and 776.4 K is 133.2 gm/dm3.
Determine the molar volume, Vm of water and the compression factor.

46. One litre of a gas at 300 atm and 473K is compressed to a pressure of 600 atm and 273 K. The
compressibility factor was found to be 1.072 and 1.375 respectively at the initial and final states.
Calculate the final volume.

CENTERS: MUMBAI / DELHI / AKOLA / KOLKATA / LUCKNOW / NASHIK / GOA #


GASEOUS ST AT E Rg - 2015 - 2017

7.1 GET EQUIPPED FOR IIT-JEE (MAIN)

Only one option is correct

1. Which of the following is not true?


(a) Density of solids is greater than gases
(b) Molecules of solid possess vibratory motion
(c) Molecules of gases possess greater kinetic energy than molecules of solid and liquid
(d) Gases like liquids possess definite volume

2. Aqueous tension of water depends upon


(a) The amount of water taken
(b) Volume of container in which water is present
(c) Temprature
(d) Some external other factors

3. Dalton’s law of partial pressures will not hold good for which of the following?
(a) H2 + O2 + CO2 (b) Cl2 + NH3 + HCl
(c) N2 + HBr + Cl2 (d) Cl2 + NH3 + O2

4. Air at sea level is dense. This can be explained by


(a) Boyle’s law (b) Charle’s law
(c) Avogadro’s law (d) Dalton’s law

5. Slope of plot between PV and P at constant temperature is


(a) 0 (b) 1 (c) 1/2 (d) 1/3

6. Steam distillation is based on


(a) Boyle’s law (b) Charle’s law
(c) Avogadro’s law (d) Dalton’s law

7. Helium gas diffuses four times faster than gas X. Molar mass of gas X is
(a) 8 (b)72 (c) 16 (d) 64

8. The average kinetic energy of which of the following gas is highest under same temperature and
pressure?
(a) H2 (b) C2H4 (c) SO2 (d)All are same

9. An LPG cylinder can withstand pressure difference of 15 atm across its boundaries. If at room temp
(270C) it is filled with 2 atm pressure. Determine the temperature at which it will explode?
(a) 2160C (b) 21270C (c) 24000C (d) 270C

10. A balloon having 8 gm Ne gas and radius 20 cm is pricked and 7 gm of Ne gas effused from it. What
will be the radius of balloon under similar conditions :
(a) 5 cm (b) 10 cm (c) 15 cm (d) 20 cm

11. A student put a reaction mixture in a round flask at 27 0C on the flame. After some time, temperature
of flask was 4770C. What fraction of air would have been expelled out?
(a) 2/5 th (b) 3/5 th (c)1/5 th (d) 4/5 th

CENTERS: MUMBAI / DELHI / AKOLA / KOLKATA / LUCKNOW / NASHIK / GOA #


GASEOUS ST AT E Rg - 2015 - 2017

12. Temperature and pressure in Delhi is 470C and 800 mmHg whereas in Mumbai is 27 0C and 750
mmHg. What is ratio of densities of air in Delhi and Mumbai?
(a) 1 : 1.63 (b) 1 : 0.957 (c)1 : 1.12 (d) 1 : 1

13. The density of a gas is 16 at NTP. At what temperature its density will be 14 if pressure remains
constant
(a) 500C (b) 390C (c) 570C (d) 430C

14. A closed flask contains water in all its three states at 0 0C. In this situation average kinetic energy of
water molecule will be
(a) Same in all the three states (b) the greatest in vapour state
(c) the greatest in liquid state (d) the greatest in solid state

15. Two flasks A and B have capacity 1 L and 2 L and each of them contains 1 mole of a gas. The
temperatures of the flasks are adjusted that average speed of molecules in A is twice as those in B.
The pressure in flask A would be
(a) Same as that in Y (b) half of that in Y
(c) twice of that in Y (d) 8 times of that in Y

16.

The velocity C1 in the above figure is given by the relation


3RT 2RT 8RT 3RT
(a) C1 = (b) C1 = (c) C1 = (d) C1 =
M M pM M

17. A closed vessel contains equal volume of N2 & O2 at a pressure of P. If N2 is removed from the
system then the pressure will be
(A) P (B) 2P (C) P/2 (D) P2

18. According to kinetic theory of gases:


(a) the pressure exerted by given mass of a gas is proportional to mean square velocity of the
molecules at constant volume.
(b) the pressure exerted by the gas is proportional to the root mean square velocity of the molecules.
(c) the root mean square velocity is inversely proportional to the temperature.
(d) the mean K.E. of the molecule is directly proportional to the temperature in degree centigrade.

19. Which of the following is incorrect statement?


(a) vander Waals’ constant ‘a’ is a measure of attractive force.
(b) vander Waals’ constant ‘b’ is also called co-volume or excluded volume per mole.
(c)‘b’ is expressed in L mol–1
(d) ‘b’ is one-fourth of critical volume

CENTERS: MUMBAI / DELHI / AKOLA / KOLKATA / LUCKNOW / NASHIK / GOA #


GASEOUS ST AT E Rg - 2015 - 2017
20. Vander waal’s constant ‘a’ and ‘b’ are related with
(a) Attractive force and bond energy of molecules respectively
(b) Volume and repulsive forces of molecules respectively
(c) Shape and repulsive forces of molecules respectively
(d) Attractive force and volume of the molecules respectively

21. A gas deviated from ideal behavior at a high pressure because its molecules
(a) have kinetic energy (b) are bound by covalent bonds
(c) have different shape (d) attract one another

22. When do you expect a real gas to behave like an ideal gas?
(a) When both the temperature and pressure are low
(b) When both the temperature and pressure are high
(c) When the temperature is high and pressure is low
(d) When the temperature is low and pressure is high

23. Dominance of strong repulsive forces among the molecules of real gas
(Z = compressibility factor)
(a) Depends on Z and indicated by Z = 1 (b) Depends on Z and indicated by Z < 1
(c) Depends on Z and indicated by Z > 1 (d) Is independent of Z

24. One mole of SO2 gas occupies a volume of 900 ml at 24 atm pressure at 270C. Than
(a) gas is showing negative deviation from ideal behaviour
(b) gas is showing positive deviation from ideal behaviour
(c) gas is showing no deviation from ideal behaviour
(d) cannot be determined

25. At critical temperature which kind of forces are dominating?


(a) Attractive forces (b) Repulsive forces (c) Both are same (d) cannot be determined
26. A gas is easily liquefied
(a) Above critical temperature and below critical pressure
(b) Below critical temperature and above critical pressure
(c) Below critical temperature and critical pressure
(d) Above critical temperature and critical pressure

27. Which of the following is incorrect statement?


(a) Ideal gases are having Z = 1 and cannot be liquefied.
(b) When Z > 1, real gases are difficult to compress
(c) When Z > 1, real gases are easier to compress
(d) When Z < 1, real gases are easier to compress

28. At the critical temperature


(a) Liquid and vapour exist in equilibrium (b) The meniscus between liquid and vapour disappears
(c) Vapour state does not exist at all (d) The vapour condense into solid
29. If for a gas critical parameters are Pc = 2 atm, Tc = 470C, then approximate value of Vc will be
(a) 10 L (b) 5 L (c) 20 L (d) 2 L

30. At low pressure vander Waal’s equation for 4 moles of a real gas will have its simplified form
PV PV
(a) PV =4 (b) PV
4a = 4 (c) =1 (d) a
=4
RT + Pb RT - RT + 4 Pb RT -
V V

7.2 GET EQUIPPED FOR IIT-JEE (ADVANCED)


CENTERS: MUMBAI / DELHI / AKOLA / KOLKATA / LUCKNOW / NASHIK / GOA #
GASEOUS ST AT E Rg - 2015 - 2017

Only one option is correct

1. The mole fraction He is 0.4 in gaseous mixture He and CH4. If both the gaseous are effusing through
the constant area of the orifice of the container, then what will be % composition by volume of CH4
gas effusing out initially?
(A) 50% (B) 40% (C) 43% (D) 75%

2. Which does not represent isobar given by Charles’ law?


(A) V (B) V

to(C) T(K)

(C) both (A) and (B) (D) none of these

3 Two gas samples are separated by a conducting frictionless piston as shown in the figure :
If the piston is in equilibrium in the position shown. Then which of the following statements is NOT
true :

(A) TA = TB (B) PA = PB
(C) nB = 3nA (D) Total K.E. of Gas A = 3× Total K.E. of gas B

4. What percent of a sample of nitrogen must be allowed to escape if its temperature, pressure and
volume are to be changed as given?
273oC, 3.00 atm 0oC, 0.75 atm
1.65 L 0.55 L
(A) 16.67% (B) 83.33% (C) 75.00% (D) 25.00%

5 In the above figure mercury columns of 10 cms each are trapped P gas = ?
between gas columns of 10 cm each. If Patm = 75 cm of Hg then the
gas pressure in the topmost column will be H g 10cm
(A) 55 cm of Hg (B) 35 cm of Hg
(C) 65 cm of Hg (D) 45 cm of Hg 10cm
H g 10cm

10cm
H g

CENTERS: MUMBAI / DELHI / AKOLA / KOLKATA / LUCKNOW / NASHIK / GOA #


GASEOUS ST AT E Rg - 2015 - 2017
6. In the quantitative determination of nitrogen, N2 gas liberated from 0.42 gm of a sample of organic
100
compound was collected over water. If the volume of N2 gas collected was ml at total pressure
11
860 mm Hg at 250 K, % by mass of nitrogen in the organic compound is
[Aq. tension at 250 K is 24 mm Hg and R = 0.08 L atm mol-1 K-1]
10 5 20 100
(A) % (B) % (C) % (D) %
3 3 3 3

7. 1 mole of N2O4 (g) at 300 K is kept in closed container under 1 atm. It is heated to 600K when 20%
mass of N2O4(g) decomposes to NO2(g). The resultant pressure is
(A) 1.2 atm (B) 2.4 atm (C) 2 atm (D) 1 atm

8. A definite amount of gaseous hydrocarbon having (carbon atoms less than 5) was burnt with
sufficient amount of O2. The volume of all reactants was 600 ml, after the explosion the volume of
the products [CO2(g) and H2O(g)] was found to be 700 ml under the similar conditions. The
molecular formula of the compound is
(A) C3H8 (B) C3H6 (C) C3H4 (D) C4H10

9. A spherical air bubble is rising from the depth of a lake where pressure is ‘P’ atm and temperature is
T’ Kelvin. The percentage increase in its radius when it comes to the free surface of lake will be.
(Assume temperature and pressure at the surface be respectively P/4 and 2T Kelvin) :
(A) 100% (B) 50% (C) 40% (D) 200%

10. Excess F2(g) reacts at 150� C and 1.0 atm pressure with Br2(g) to give a compound BrFn. If 423 mL
of Br2(g) at the same temperature and pressure produced 4.2 g of BrFn. what is n ? (Br = 80, F = 19)
(A) 3 (B) 1 (C) 5 (D) 7

11. The system shown in the diagram is at equilibrium at 27 o C and volume of the bulb is 150 mL. At
this temperature, the vapour pressure of water is 28 millimetres of mercury. If the bulb contains
0.001 mol of O2(g), volume of the liquid water is approximately

Vacuum

O2(g)
water

161 mm

closed end manometer

Hg

(A) 34 mL (B) 10 mL (C) 30 mL (D) 60 mL

CENTERS: MUMBAI / DELHI / AKOLA / KOLKATA / LUCKNOW / NASHIK / GOA #


GASEOUS ST AT E Rg - 2015 - 2017
12. In the following figure, when the two stop corks are opened, the total pressure inside the flask will be

CO2
Ar
2.13 atm
1.15 atm
1.5 L H2
2.0 L
1.0 L
0.861atm

(A) 1.41atm (B) 2.41atm (C) 3.41atm (D) 1.12atm

13. To determine the molar mass of a liquid, as small sample of the liquid was vaporized in a glass vessel
with a pinhole opening. The volume of the vessel was 115.3mL, and the liquid was completely
vaporized in a boiling water bath at 100�C . The atmospheric pressure that day was 743torr. After the
vapour was cooled to 25� C and had condensed back to a liquid, it was weighed. The mass of the
liquid was 0.4429g. What is the molar mass of the liquid, in units of g/mol?
(A) 120.3 (B) 271.5 (C) 16.03 (D) 96.15

14. For two gases, A and B with molecular weights MA and MB, it is observed that a certain
temperature, T, the mean speed of A is equal to the root mean square speed of B. Thus the mean
speed of A can be made equal to the mean speed of B, if:
(A) A is at temperature T and B at T¢, T > T¢
8
(B) A is lowered to a temperature T2 = T while B is at T

(C) Both A and B are raised to a higher temperature
(D) Both A and B are placed at lower temperature

15. Partial pressure are simply additive for ideal gases. For non–ideal gases (such as ammonia and
oxygen), if we tried to consider them as ideal gases, we would
(A) always overestimate the total pressure
(B) always underestimate the total pressure
(C) underestimate p at low pressure and overestimate p at high pressure
(D) overestimate p at low pressure and underestimate p at high pressure

16. A capillary tube of uniform diameter contains gas samples A and B, separated by a short column of
Hg, ‘ l ’ mm in length. The ends are sealed. In horizontal position, the confined gases occupy ‘a’ mm
and ‘b’ mm in length with a common pressure (P). In vertical position, the lengths become
respectively a ' mm and b ' mm. Then P is equal to
l l
�b a � �a b � �b a � �a b �
� - � � - � l� - � l� - �
(A) �b ' a ' �(B) �b ' a ' �(C) �b ' a ' � (D) �a ' b ' �

17 For 10 minutes each, at 00C from two identical holes nitrogen and a unknown gas are leaked into a
common vessel of 3lit capacity under identical conditions. The resulting pressure is 5.6 atm and the
mixture contains 0.5 mole of nitrogen. Then the molar mass of the unknown gas is :
(A) 224 g mol–1 (B) 112 g mol–1 (C) 400 g mol–1 (D) 540 g mol–1

CENTERS: MUMBAI / DELHI / AKOLA / KOLKATA / LUCKNOW / NASHIK / GOA #


GASEOUS ST AT E Rg - 2015 - 2017

One or more than one option correct

1 Following graph is constructed for the fixed amount of the gas.

(A) From 1 - 2 pressure will increase (B) From 2 - 3 pressure remains constant
(C) Gas pressure at (3) is greater at state (1) (D) From 1 - 2 pressure will decrease

2. Which of the following statement are incorrect?


(A) Molar volume of every ideal gas at STP is 22.4 L
(B) Under critical state compressibility factor is 1
(C) All ideal gases will have equal value of average KE at a given temperature
3
(D) At absolute zero, KE is R.
2
3. At 47oC and 16.0 atm, the molar volume of NH 3 gas is about 10% less than the molar volume of an
ideal gas. This is due to:
(A) NH3 decomposes to N2 and H2 at 47oC
(B) the force of attraction between NH3 molecules is significant
(C) the volume occupies by NH3 molecules themselves is a significant fraction of the volume of the
container at this temperature and pressure.
(D) at 16 atm, NH3 molecules no longer move randomly

4. Rate of effusion of an ideal gas through a hole depends upon


(A) the pressure of the gas (B) the molecular weight of the gas
(C) surface area of the hole (D) shape of the hole

5. The time taken for effusion of 32mL of oxygen will be the same as the time taken for
effusion under identical conditions of
(A) 64mL ofH2 (B) 50mL of N2 (C) 27.3mL of CO2 (D) 22.62mL of SO2
6. The diagram displays the plot of the compression factor Z versus p for a few gases.
Which of the following statement(s) is (are) correct for a van der Waals’ gas?

I IV

III

II
p

(A) The plot I is applicable provided the van der Waals’ constant a is negligible.
(B) The plot II is applicable provided the van der Waals’ constant b is negligible.

CENTERS: MUMBAI / DELHI / AKOLA / KOLKATA / LUCKNOW / NASHIK / GOA #


GASEOUS ST AT E Rg - 2015 - 2017
(C) The plot III is applicable provided the van der Waals’ constant a and b are negligible.
(D) The plot IV is applicable provided the temperature in the gas is much higher than its
critical temperature.

7. Knowing that average kinetic energy of an ideal gas (X) is directly proportional to
absolute temperature, if T1 = 273K, which statement(s) describe(s) the other curves?

Fraction of molecules
at a given velocity

T1(x)
(A) Curve A is for heavier gas but at same temperature
(B) Curve B is for the same gas but at 373K
(C) Curve A is for the same gas but at 373K
(D) Curve B is for lighter gas but at same temperature
8. Which of the following are wrongVelocity
prediction regarding a van der Waals’ gas?
(A) A gas with large ‘a’ has high critical temperature.
(B) A gas with large ‘b’ is more compressible.
(C) A gas with large ‘b’ has high critical pressure.
(D) A gas with large ‘a’ is easily liquefiable.

9. A gas can be liquefied


(A) above critical temperature (B) under adiabatic condition
(C) under pressure when it is cooled below critical temperature
(D) at pressure higher than the critical pressure and temperature lower than the critical temperature

10. Which of the following statements regarding compression factor is/are correct?
(A) Z for an ideal gas is independent of temperature and pressure.
(B) Z for an ideal gas is greater than one.
(C) Z for a non–ideal gas can be greater than or less than unity depending on T and P.
(D) When Z<1, intermolecular attraction dominates over intermolecular repulsion.

11. The correct statements regarding various types of molecular speeds is/are
(A) Increasing temperature increases the fraction of molecules possessing Umps
(B) Increasing temperature increases Umps.
(C) In a sample of gas, at a given temperature molecules with extremely low and extremely high
speeds are both very less likely.
(D) At the same temperature, lighter gases have narrow distribution of molecular speeds than the
heavier gases.

A C
Z
12. The given graph represents variation of (compressibility factor) versus p, for three real gases A, B
and C. Which of1the following statement(s) is/an correct?

B
CENTERS: MUMBAI / DELHI / AKOLA / KOLKATA / LUCKNOW / NASHIK / GOA #

p
GASEOUS ST AT E Rg - 2015 - 2017

(A) For the gas A, a = 0 and its dependence on p is linear at all pressures.
(B) For the gas B, b = 0 its dependence on P is linear at all pressures.
(C) For the gas C, which is a typical real gas for which neither ‘a’ nor ’b’ is zero. By knowing the
minima and point of intersection, with Z = 1, ‘a’ and ‘b’ can be determined.
(D) At high pressure, slope is positive for all gases.
13. When gas is expanded at constant temperature
(A) pressure decreases
(B) the average kinetic energy of molecules remain the same
(C) the average kinetic energy decreases
(D) the average kinetic energy increases

14. A mixture of 100 ml CO, CO 2 and O2 was sparked. When the resulting gaseous mixture was passed
through KOH solution, contraction in volume was found to be 80 ml, the composition of initial
mixture may be (in the same order)
(A) 30 ml, 60 ml, 10ml (B) 30 ml, 50ml, 20ml
(C) 50 ml, 30 ml, 20ml (D) 30ml, 40ml, 30ml

15. An organic compound is burnt with excess of O2 to produce CO2(g) and H2O(l), which results in 25%
volume contraction. Which of the following option(s) satisfy the given conditions.
(A) 10 ml C3H8 + 110 ml O2 (B) 20 ml C2H6 O + 80 ml O2
(C) 10 ml C3H6O2 + 50 ml O2 (D) 40 ml C2H2O4 + 60 mlO2

16. A mixture of C3H8 (g) & O2 having total volume 100 ml in an Eudiometry tube is sparked & it is
observed that a contraction of 45 ml is observed. What can be the composition of reacting mixture?
(A) 15 ml C3H8 & 85 ml O2 (B) 25 ml C3H8 & 75 ml O2
(C) 45 ml C3H8 & 55 ml O2 (D) 55 ml C3H8 & 45 ml O2

COMPREHENSION TYPE
PASSAGE−1
The behaviors of gases has been expressed in terms of various gas laws obtained
experimentally i.e. Boyle’s law, Charles’ law Pressure – temperature law, Dalton’s law of partial
pressure, Graham’s law of diffusion and Avogadro’s hypothesis. Later on Maxwell derived kinetic
1
equation PV = mnu , theoretically by assuming the concept of molecules and their motion. The
2

3
terms u represents root mean square speed of molecule.

�d P ��d V ��d V �
1. For one mole of an ideal gas � �� �� �is equal to:
�d T ��
V d T ��
P dP �T

CENTERS: MUMBAI / DELHI / AKOLA / KOLKATA / LUCKNOW / NASHIK / GOA #


GASEOUS ST AT E Rg - 2015 - 2017

R2 R2
(A) - (B) (C) + 1 (D) – 1
P2 P2

2. The intercept of plots of log V vs log T curves at constant pressure P for 1 mole gas will
be :
P -P R -R
(A) log (B) log (C) log (D) log
R R P P

3. If intramolecular forces vanishes among water molecules, then volume occupied by 1.8 g H 2O (l) at
STP is :
(A) 2.24 litre (B) 22.4 litre (C) 1.8 mL (D) 2.24 mL

4. The urms of SO2 at 27o C is:


(A) 3.42 x 102 m sec – 1 (B) 10.82 m sec -1
(C) 11.69 x 10 4 m sec -1 (D) none of these

5. The root mean square speed of gas at 300 K is 3 R . The molar mass of gas in kg/mol is
(A) 1 (B) 10 (C) 100 (D) 1,000

PASSAGE−2
On the recently discovered 10th planet it has been found that the
gases follow the relationship PeV 2 = nCT , where C is a constant, other
notation are as usual (V in lit., P in atm and T in Kelvin). A curve is
plotted between P and V at 500 K & 2 moles of gas as shown in figure:

1. The value of constant C is


(A) 0.01 (B) 0.001 (C) 0.005 (D) 0.002

2. Find the slope of the curve plotted between P Vs T for closed container of volume 2 lit. having same
moles of gas
e 2
(A) (B) 2000e (C) 500e (D)
2000 1000e

3. If a closed container of volume 200 lit. of O 2 gas (ideal gas) at 1 atm & 200 K is taken to planet. Find
the pressure of oxygen gas at the planet at 821 K in same container
10 20
(A) atm (B) atm (C) 1 atm (D) 2 atm
e100 e50

PASSAGE−3
A gas present in a container connected to frictionless, weightless piston acting as a sieve operating
always at one atmosphere pressure such that it permits flow of gas outside (with no adding of gas).
The graph of n (No. of moles) vs T (Kelvin) was plotted & was found to be a straight line with
co-ordinates of extreme points as (300, 2) & (200, 3).
1 atm

(200, 3)

Number
of
moles

(300, 2)
n
T
Temperature(K)

CENTERS: MUMBAI / DELHI / AKOLA / KOLKATA / LUCKNOW / NASHIK / GOA #


GASEOUS ST AT E Rg - 2015 - 2017

1. The best fit relationship between n & T:


100 600 T T
(A) T = - +5 (B) n = (C) n = 5 - (D) n = -5 +
n T 100 100

2. The best fit relationship between V & T: (Assuming gas inside is ideal gas)
RT 2 - RT 2 - RT 3 RT 2
(A) V = - RT (B) V = + 5RT (C) V = + 5RT 2 (D) V =
100 100 200 100

3. Calculate the temperature at which height of piston is maximum or minimum:


(A) 127oC (B) -23oC (C) 227oC (D) -123oC

4. Calculate the maximum volume of occupied by the gas


(A) 50 R (B) 125 R (C) 50.5 R (D) 625 R

PASSAGE−4
First complete data on pressure - volume -temperature relations
of a substance in both gaseous and liquid state was obtained by
Thomas Andrews on carbon dioxide. He plotted isotherms of
carbon dioxide at various temperatures. Later on it was found 60oC
that real gases behave in the same manner as carbon dioxide. 50oC
Andrews noticed that at high temperatures isotherms look like P

that of an ideal gas and the gas cannot be liquefied even at very 31.1oC

high pressure. As the temperature is lowered, shape of the curve 20oC


changes and data shows considerable deviation from ideal 13oC
behaviour. V

1. From the given graph the critical temperature of CO2 is


(A) 50oC (B) 31.1oC (C) 20oC (D) 13oC

2. The critical temperatures for gases A, B, C and D are 126.0K, 154.3K, 304.15K and 405.5K
respectively. Which gas shows greater negative deviation from ideal behavior with increase in
pressure?
(A) A (B) B (C) C (D) D

3. The compressibility factor of a Vander Walls gas at its critical point is


(A) 1 (B) 0.375 (C) 2.67 (D) -1

PASSAGE−5
The apparatus shown consists of three temperature jacketed 1.000 L bulbs connected by stopcocks.
Bulb A contains a mixture of H2O(g), CO2(g) and N2(g) at 250C and a total pressure of 564 mm Hg.
Bulb B is empty and is held at a temperature of –700C. Bulb C is also empty and is held at a
temperature of
–1900C. The stopcocks are closed, and the volume of the lines connecting the bulb is zero. CO 2
sublimes at –780C, and N2 boils at –1960C.

CENTERS: MUMBAI / DELHI / AKOLA / KOLKATA / LUCKNOW / NASHIK / GOA #


GASEOUS ST AT E Rg - 2015 - 2017

1. The stopcock between A and B is opened and the system is allowed to come to equilibrium. The
pressure in A and B is now 219 mm Hg. Select the correct alternate:
(A) A contains CO2(g) and N2(g) and B contains CO2(g), N2(g) and H2O(s)
(B) A contains CO2(g) and B contains N2(g) and H2O(l)
(C) A contains CO2(g), N2(g) and H2O(s) and B contains CO2(g) and N2(g)
(D) A contains H2O(g) and B contains H2O(g), N2(g) and CO2(g)

2. How many moles of H2O are in the system?


(A) 0.026 mol (B) 0.0013 mol (C) 0.013 mol (D) 0.13 mol

3. Both stopcocks are opened and the system is again allowed to come to equilibrium. The pressure
throughout the system is 33.5 mm Hg. Select the correct alternate.
(A) Each bulb contains N2 (g), H2O(s) and CO2(g)
(B) Each bulb contains N2(g), H2O(g) and CO2(g)
(C) Each bulb contains N2(g), H2O(s) and CO2(s)
(D) A contains N2(g), B contains N2(g), H2O(s); C contains N2(g) and CO2(s)

4. How many moles of N2 are in the system?


(A) 0.022 mol (B) 0.011 mol (C) 0.018 mol (D) 0.036 mol

5. How many moles of CO2 are in the system?


(A) 0.022 mol (B) 0.011 mol (C) 0.018 mol (D) 0.036 mol

PASSAGE−6
The constant motion and high velocities of gas particles lead to some important practical
consequences. One such consequence is that gases mix rapidly when they come in contact. Take the
stopper off a bottle of perfume, for instance, and the odour will spread rapidly through the room as
perfume molecules mix with the molecules in the air. This mixing of different gases by random
molecular motion and with frequent collisions is called diffusion. A similar process in which gas
molecules escape without collision through a tiny hole into a vacuum is called effusion.

1. When CO2 under high pressure is released from a fire extinguisher, particles of solid CO 2 are
formed despite the low sublimation temperature (-770C) at 1 atm. It is due to:
(A) The gas does work pushing back the atmosphere using kinetic energy of molecules and thus
lowering the temperature.
(B) Volume of the gas is decreased rapidly hence temperature is lowered.
(C) Both of the above
(D) None of the above

2. The stopcocks of the bulbs X (containing NH 3) and Y (containing HCl) are opened simultaneously;
white fumes of NH4Cl and formed at point B. If AB = 36.5 cm, then BC is approximately:

CENTERS: MUMBAI / DELHI / AKOLA / KOLKATA / LUCKNOW / NASHIK / GOA #


GASEOUS ST AT E Rg - 2015 - 2017
(A) 18.0 cm (B) 25.0 cm (C) 20.0 cm (D) 36.5 cm

PASSAGE: 7
A sealed flask 1 contains two different gases A and B and their root mean square speeds are 500 and
400 metre per second respectively. Now this flask is connected to another flask 2, containing only A
with their root mean square speed 400 ms–1, by means of a narrow tube of negligible volume and
gases in the two flask were allowed to mix up.

1. In the combined system


(A) A will be moving at rms less than 500 ms–1, but B will continue moving at its original rms.
(B) rms of A will increase but rms of B will decrease.
(C) rms of both A and B will decrease below to 400 ms–1
(D) rms of A will decrease below 500 ms–1 and rms of B will decrease below 400 ms–1
2. Which of the following statement regarding molar masses of A and B is true ?
(A) Molar mass of A is greater than molar mass of B
(B) Molar mass of B is greater than molar mass of A.
(C) Molar mass can’t be compared unless temperature is specified
(D) Molar mass of either A or B can be greater or less than one another depending on their atomicity
3. Which of the following statement is true regarding partial pressures of A and B ?
(A) Partial pressure of A in the combined system must be equal to its partial pressure in flask 1.
(B) Partial pressure of A is more in flask 1 than its partial pressure in the combined system.
(C) Partial pressure of B in the combined system must be less than its partial pressure in flask 1.
(D) Final total pressure is less than the total pressure in flask 1.

PASSAGE 8
When air from an air balloon is replaced by gases like H2, He, CH4 etc, gas filled balloon rises from the
surface. If the air is replaced by any of the above mentioned gas maintaining same pressure, volume
and temperature, the maximum weight that the gas filled balloon can lift is known as “ Payload” of
balloon and is determined as
Payload = Weight of air – Weight of gas – Weight of balloon

1. What is the most appropriate reason for the rise of gas filled balloon ?
(A) Lighter gas will have higher kinetic energy than the air molecule.
(B) Lighter gas are more likely to behave like an ideal gas
(C) Lower density of gas in gas filled balloon than air in air filled balloon.
(D) Air is mixture of several gases.

2. Which of the following gas when filled in a balloon will rise to the same height as a He–gas filled
under identical condition of P, V and T ?
(A) H2(g) (B) D2(g) (C) Ne(g) (D) CH4(g)

3. If a massless balloon is filled to 100 L with He gas at 300K and one atm, the payload of this balloon
will be (in gram unit)? Assume air to be a mixture of 80% N 2(g) and 20% O2(g) by volume and
atmospheric condition is 1.0 atm and 300 K.
(a) 50 (b) 100 (c) 134 (d) 17

PASSAGE – 9
A 10 ml mixture of N2, alkane & O2 undergo combustion in the Eudiometry tube. When the residual
gases were passed through KOH, a contraction of 2ml was observed. To the remaining mixture

CENTERS: MUMBAI / DELHI / AKOLA / KOLKATA / LUCKNOW / NASHIK / GOA #


GASEOUS ST AT E Rg - 2015 - 2017
(comprising of only one gas) excess H2 was added & after the reaction the gas produced was absorbed
by water, causing a reduction in volume of 8 ml.
1. What should be the gas produced after introduction of H2 in the mixture?
(A) H2O (B) CH4 (C) CO2 (D) NH3
2. What is the volume of N2 present in the mixture?
(A) 2 ml (B) 4 ml (C) 6 ml (D) 8 ml

3. What is the volume of O2 remained after the first combustion?


(A) 4 ml (B) 2 ml (C) 0 ml (D) 8 ml

4. Identify the hydrocarbon.


(A) CH4 (B) C2H6 (C) C3H8 (D) C4H10

MATCHING TYPE

1. Match the gases with the associated conditions in the left column with their corresponding properties in the right column.
Column I Column II
H2{p = 200 atm, T = 273K} Z �1
H2 [p ~ 0, T = 273] Attractive force dominating
CO2(g) [p = 1 atm, T = 273] pV = nRT
CO2(g) [p = 200 atm, T = 273] Repulsive force dominating

2. Match the properties in the left column with corresponding functions in the right column.
Column I Column II
Compression factor (Z) of 8a
ideal gas 27 Rb
Z for real gas at low P pb
1+
RT
Z for real gas at high pressure 1.00
Critical temperature (Tc) a
1-
VRT

CENTERS: MUMBAI / DELHI / AKOLA / KOLKATA / LUCKNOW / NASHIK / GOA #


GASEOUS ST AT E Rg - 2015 - 2017

3. Match the kinetic properties in the left column with their corresponding expression in the right column.
Column I Column II
RMS speed 8RT
pM
Average speed 1 mn(c) 2
3 V
Most probable speed 3RT
M
Pressure by kinetic theory 2RT
M

4.
Column – I Column – II (In a closed rigid vessel)
Density of gas Depends on absolute temperature.
Average KE of gas Depends on molecular mass of gas.
Root mean square speed Depends on total molecules of gas
Effusion rate of gas Independent of molecular mass of gas

CENTERS: MUMBAI / DELHI / AKOLA / KOLKATA / LUCKNOW / NASHIK / GOA #


GASEOUS ST AT E Rg - 2015 - 2017

8.1 WINDOW TO IIT-JEE (MAIN)

1. Value of gas constant R is : [AIEEE - 2001]


(A) 0.082 litre atm. (B) 0.987 cal mol –1 K–1 –1
(C) 8.3 J mol K –1
(D) 83 erg mol–1 K–1

2. Kinetic theory of gases proves : [AIEEE - 2002]


(A) Only Boyle’s law (B) Only Charle’s law
(C) Only Avogardro’s law (D) All of these

3. For an ideal gas, number of moles per litre in terms of its pressure P, gas constant R and
Temperature T is: [AIEEE - 2002]
(A) PT/R (B) PRT (C) P/RT (D) RT/P

4. According to kinetic theory of gases in an ideal gas between two successive collisions a gas
molecule travels: [AIEEE - 2003]
(A) In a straight line path (B) With an accelarated velocity
(C) In a circular path (D) In a wavy path

5. What volume of hydrogen gas, at 273 K and 1 atm pressure will be consumed in obtaining 21.6g
of elemental boron (atomic mass = 10.8) from the reduction of boron trichloride by hydrogen?
[AIEEE - 2003]
(A) 89.6 L (B) 67.2 L (C) 44.8 L (D) 22.4 L

6. As the temperature is raised from 20oC to 40oC, the average kinetic energy of neon atoms
Changes by a factor : [AIEEE - 2004]
(A) 2 (B) 313 (C) 313 (D) 1
293 293 2

7. In vander Waal’s equation of state of the gas law, the constant ‘b’ is a measure of :[AIEEE - 2004]
(A) Intermolecular collisions per unit volume (B) Intermolecular attractions
(C) Volume occupied by the molecules (D) Intermolecular repulsions

8. Which one of the following statements is not true about the effect of an increase in temperature
on the distribution of molecular speeds in a gas ? [AIEEE - 2005]
(1) The area under the distribution curve remains the same as under the lower temperature
(2) The distribution becomes broader
(3) The fraction of the molecules with the most probable speed increases
(4) The most probable speed increases

CENTERS: MUMBAI / DELHI / AKOLA / KOLKATA / LUCKNOW / NASHIK / GOA #


GASEOUS ST AT E Rg - 2015 - 2017

9. Phosphorus pentachloride dissociates as follows, in a closed reaction vessel,


PCl5 (g) PCl3 (g) Cl2 (g)
If total pressure at equilibrium of the reaction mixture is P and degree of dissociation of PCl5 is
x, the partial pressure of PCl3 will be [AIEEE - 2006]
�x � �2x � �x � �x �
(A) � �P (B) � � P (C) � � P (D) � �P
1+ x �
� �1- x � �x - 1 � 1- x �

10. Equal masses of methane and oxygen are mixed in an empty container at 25°C. The fraction of the
[AIEEE – 2007]
total pressure exerted by oxygen is

(A) 1/2 (B) 2/3 (C) (1/3) X (273/298) (D) 1/3.

–4 3 3

11. If 10 dm of water is introduced into a 1.0 dm flask to 300 K, how many moles of water are in
2
the vapour phase when equilibrium is established ? (Given : Vapour pressure of H O at 300 K is 3170
–1 –1

Pa ; R = 8.314 J K mol ) [AIEEE – 2010]


–3 –2 –2 –3

(A) 5.56 x 10 mol (B) 1.53 x 10 mol (C) 4.46 x 10 mol (D) 1.27 x 10 mol

CENTERS: MUMBAI / DELHI / AKOLA / KOLKATA / LUCKNOW / NASHIK / GOA #


GASEOUS ST AT E Rg - 2015 - 2017

8.2 WINDOW TO IIT-JEE (ADVANCED)

Objective Questions I [Only one correct option]

1. The ratio of root mean square to average velocity of a gas molecule at a particular temperature is
(JEE 1981)
(a) 1.086 : 1 (b) 1:1.086 (c) 2:1.086 (d) 1.086:2

2. The temperature at which a real gas obeys the ideal gas laws over a wide range of pressure is
(JEE 1981)
(a) Critical temperature (b) Boyle temperature
(c) inversion temperature (d) reduced temperature

3. Equal weights of methane and oxygen are mixed in an empty container at 25°C. The fraction of the
total pressure exerted by oxygen is (JEE 1981)
1 1 2 1 273
(a) (b) (c) (d) �
3 2 3 3 298

4. Helium atom is two times heavier than a hydrogen molecule. At 298 K, the average kinetic energy of a
helium atom is (JEE 1982)
(a) two times that of a hydrogen molecule (b) same as that of a hydrogen molecule
(c) four times that of a hydrogen molecule (d) half that of a hydrogen molecule

5. When an ideal gas undergoes unrestrained expansion, no cooling occurs because the molecules
(JEE 1984)
(a) are above the inversion temperature (b) exert no attractive forces on each other
(c) do work equal to loss in kinetic energy (d) collide without loss of energy

6. Equal weight of methane and hydrogen are mixed in an empty container at 25°C. The fraction of the
total pressure exerted by hydrogen is (JEE 1984)
1 8 1 16
(a) (b) (c) (d)
2 9 9 17

7. A liquid is in equilibrium with its vapour at its boiling point. On the average, the molecules in the two
phases have equal (JEE 1984)
(a) inter – molecular force (b) potential energy
(c) kinetic energy (d) total energy

8. Rate of diffusion of a gas is (JEE 1985)


(a) directly proportional to its density
(b) directly proportion to its molecular weight
(c) directly proportional to the square root of its molecular weight
(d) inversely proportional to the square root of its molecular weight

9. The average velocity of an ideal gas molecular at 27° C is 0.3 m/s. The average velocity at 927° C will
be (JEE 1986)
(a) 0.6m/s (b) 0.3m/s (c) 0.9m/s (d) 3.0m/s

10. In van der Waals equation of state for a non-ideal gas, the term that accounts for intermolecular forces is
(JEE 1988)
CENTERS: MUMBAI / DELHI / AKOLA / KOLKATA / LUCKNOW / NASHIK / GOA #
GASEOUS ST AT E Rg - 2015 - 2017

� a �
(a) (V-b) (b) RT (c) �p + 2 � (d) (RT)-1
� V �

11. A bottle of dry ammonia and a bottle of dry hydrogen chloride connected through a long tube are
opened simultaneously at both ends the white ammonium chloride ring first formed will be
(JEE 1988)
(a) at the centre of the tube (b) near the hydrogen chloride bottle
(c) near the ammonia bottle (d) throughout the length of the tube

12. The value of van der Waals’ constant ‘a’ for the gases O 2, N2 , NH3 and CH4 are 1.360 , 1.390, 4.170 and
2.253 L2 atm mol-2 respectively. The gas which can most easily be liquefies is
(JEE 1989)
(a) O2 (b) N2 (c) NH3 (d) CH4

13. The density of neon will be highest at (JEE 1990)


(a) STP (b) 0° C, 2atm (c) 273° C,1 atm (d) 273°C, 2 atm

14. The rate of diffusion of methane at a given temperature is twice that of a gas X. The molecular weight
of X is (JEE 1990)
(a) 64.0 (b) 32.0 (c) 4.0 (d) 8.0

15. According to kinetic theory of gases, for a diatomic molecule (JEE 1991)
(a) the pressure exerted by the gas is proportional to mean velocity of the molecule
(b) The pressure exerted by the gas is proportional to the root mean velocity of the molecule
(c) The root mean square velocity of the molecule is inversely proportional to the temperature
(d) the mean translational kinetic energy of the molecule is proportional to the absolute temperature

16. At constant volume, for a fixed number of moles of a gas the pressure of the gas increases with rise of
temperature due to (JEE 1992)
(a) increases in average molecular speed
(b) increases rate of collisions amongst molecules
(c) increase in molecular attraction
(d) decrease in mean free path

17. Equal weights of ethane and hydrogen are mixed in an empty container at 25°C the fraction of the total
pressure exerted by hydrogen is (JEE 1993)
(a) 1:2 (b) 1:1 (c) 1:16 (d) 15:16

18. The ratio between the root mean square speed of H2 at 50 K and that of O2at 800 K is (JEE 1996)
1
(a) 4 (b) 2 (c) 1 (d)
4

19. The compressibility factor for an ideal gas is (JEE 1997)


(a) 1.5 (b) 1.0 (c) 2.0 (d) ∞

20. The critical temperature of water is higher than that of O2 because the H2O molecule has
(JEE 1997)
(a) fewer electrons thanO2 (b)two covalent bonds
(c) V-shape (d) dipole moment

CENTERS: MUMBAI / DELHI / AKOLA / KOLKATA / LUCKNOW / NASHIK / GOA #


GASEOUS ST AT E Rg - 2015 - 2017
rA
21. According to Graham’s law, at a given temperature the ratio of the rates of diffusion of gases A and
rB
B is given by (where, p and M are pressures and molecular weights of gases A and B respectively)
(JEE 1998)
1 1 1 1

(a) �p A �
�M A � (b) �M A �
�p A � (c) �p A �
�M B � (d) �M A �
�pB �
2 2 2 2

� � � � � � � � � � � � � � � �
�pB ��M B � �M B �
�pB � �pB ��M A � �M B �
�p A �

22. A gas will approach ideal behaviour at (JEE 1999)


(a)low temperature and low pressure (b)low temperature and high pressure
(c) high temperature and low pressure (d) high temperature and high pressure

23. The rms velocity of hydrogen is 7 times the rms velocity of nitrogen. If T is the temperature of the
gas (JEE 2000)
(a) T (H2) = T(N2) (b) T(H2) > T(N2) (c) T(H2) <T(N2) (d) T(H2) = 7 T(N2)

24. The compressibility of a gas is less than unity at STP. Therefore, (JEE 2000)
(a) Vm> 22.4L (b) Vm< 22.4 L(c) Vm = 22.4L (d) Vm = 44.8L

25. At 100°C and 1 atm if the density of the liquid water is 1.0g cm -3 and that of water vapour is 0.0006 g
cm-3 , then the volume occupied by water molecules in 1 L of steam at this temperature is
(JEE 2000)
(a) 6cm3 (b) 60cm3 (c) 0.6cm3 (d) 0.06cm3

26. The root mean square velocity of an ideal gas at constant pressure varies with density (d) as
(JEE 2001)
2
(a) d (b) d (c) d (d) 1/ d

27. Which of the following volume (V) temperature (T) plots represents the behaviour of one mole of an
ideal gas at the atmospheric pressure? (JEE 2002)
V(L) (38.8 L, V(L) (28.6 L,
373K) 373K)
(2 2 .4 L , (2 0 .4 L ,
273K ) 273K )
T(K) T(K)
(a) (b)
V(L) (30.6 L, V(L)
373K)
(2 2 .4 L , (2 2 .4 L ,
273K ) (14.2 L,
273K ) 373K)
T(K) T(K)
(c) (d)

28. Positive deviation from ideal behaviour takes place because of (JEE 2003)
(a) molecular interaction between atom and pV/nRT>1
(b) molecular interaction between atom and pV/nRT < 1
(c) finite size of atoms and pV/nRT > 1
(d) finite size of atom and pV/nRT < 1.

CENTERS: MUMBAI / DELHI / AKOLA / KOLKATA / LUCKNOW / NASHIK / GOA #


GASEOUS ST AT E Rg - 2015 - 2017
29. For a monatomic gas kinetic energy = E. The relation with rms velocity is (JEE 2004)
1/2 1/2 1/2 1/2
(a) u = �2E � (b) u = �3E � (c) u = �E � (d) u = �E �
� � � � � � � �
�m � �2m � �2m � �3m �

30. If helium and methane are allowed to diffuse out of the container under the similar conditions of
temperature and pressure, then the ratio of rate of diffusion of helium to methane is (JEE 2009)
(a) 2.0 (b) 1.0 (c) 0.5 (d) 4.0

31. The term that corrects for the attractive forces present in a real gas in the van der Waals’ equation is
(JEE 2009)
2 2 2 2
(a) nb (b) n a/V (c) –(n a/V ) (d) -nb

Objective Questions II (one or more than one correct option)


0
1. If a gas is expanded at constant temperature (JEE 1986)
(a) the pressure decreases
(b) the kinetic energy of the molecules remains the same
(c) the kinetic energy of the molecules decreases
(d) the number of molecules of the gas increases
pV
2. The given graph represent the variations of Z (compressibility factor (Z) = ) versus p, for three real
nRT
gases A, B and C. (JEE 2006)

C
A
A
Ideal gas
1 B
Z C
B
O P ( atm )
Identify the only incorrect statement
(a) For the gas A, a=0 and its dependence on p is linear at all pressure.
(b) For the gas B, b = 0 and its dependence on p is linear at all pressure.
(c) For the gas C, which is typical real gas for which neither a nor b = 0. By Knowing the minima and
the point of interaction, with Z = 1, a and b can be calculated.
(d) At high pressure, the slope is positive for all real gases.

3. A gas described by van der Waals equation (JEE 2008)


(a) behaves similar to an ideal gas in the limit of large molar volumes
(b) behaves similar to an ideal gas in the limit of large pressures
(c) is characterized by van der Waals coefficients that are dependent on the identity of the gas but are
independent of the temperature
(d) has the pressure that is lower than the pressure exerted by the same gas behaving ideally.

4. According to kinetic theory of gases (JEE 2011)


(a) collisions are always elastic
(b) heavier molecules transfer more momentum to the wall of the container
(c) only a small number of molecules have very high velocity
(d) between collisions, the molecules move in straight lines with constant velocities
Assertion and Reason

CENTERS: MUMBAI / DELHI / AKOLA / KOLKATA / LUCKNOW / NASHIK / GOA #


GASEOUS ST AT E Rg - 2015 - 2017
Read the following questions and answer as per the direction given below:

(a) Statement I is true ; Statement II is true; Statement II is the correct explanation of Statement I.
(b) Statement I is true; Statement II is true; Statement II is not the correct explanation of Statement I.
(c) Statement I is true; Statement II is false.
(d) Statement I is false; Statement II is true.

1. Statement I : The value of Van der Waal’s constant ‘a’ is larger for ammonia than for nitrogen.
Statement II: Hydrogen bonding is present in ammonia. (JEE 1998)

2. Statement I: The pressure of a fixed amount of an ideal gas is proportional to its temperature.
Statement II : Frequency of collisions and their impact both increase in proportion to the square root of
temperature. (JEE 2000)

Match the columns


1. Match the gases under specified conditions listed in Column I with their properties / laws in Column
(JEE 2007)
Column I Column II
(A) Hydrogen gas (p = 200 atm, T = 273 K) (p) compressibility factor ≠1
(B) Hydrogen gas (p~0 atm, T = 273 K) (q) attractive forces are dominant
(C) CO2 (p=1 atm, T = 273 K) (r) pV = nRT
(D) Real gas with very large molar volume (s) p(V-nb) = nRT

Fill in the Blanks:

1. The total energy of one mole of an ideal monatomic gas at 27°C is _____cal. (JEE 1984)
2. Cp– Cv for an ideal gas is ______. (JEE 1984)
3. The rate of diffusion of a gas is _____ proportional to both ____ and square root of molecular mass.
(JEE 1986)
4. The value of pV for 5.6 L of an ideal gas is ____ RT, at NTP (JEE 1987)
5. 8 g each of oxygen and hydrogen at 27° C will have the total kinetic energy in the ratio of _____.
(JEE 1989)
6. The absolute temperature of an ideal gas is ____ to/ than the average kinetic energy of the gas
molecules. (JEE 1997)

True/ False

1. Kinetic energy of a molecule is zero at 0°C. (JEE 1985)

2. A gas in a closed container will exert much higher pressure due to gravity at the bottom than at the top.
(JEE 1985)
2
� n a�
3. In the van der Waals’ equation, �p + 2 � (V - nb) = nRT
� V �

The constant ‘a’ reflects the actual volume of the gas molecules. (JEE 1993)

4. A mixture of ideal gases is cooled up to liquid helium temperature (4.22K) to form an ideal solution.
(JEE 1996)

Integer type Questions

CENTERS: MUMBAI / DELHI / AKOLA / KOLKATA / LUCKNOW / NASHIK / GOA #


GASEOUS ST AT E Rg - 2015 - 2017

1. To an evacuated vessel with movable piston under external pressure of 1 atm 0.1 mole of He and 1.0
mole of an unknown compound (vapour pressure 0.68 atm at 0° C) are introduced. Considering the
ideal gas behaviour, the total volume (in litre) of the gases at 0° C is close to
(JEE 2011)
Subjective Questions:

1. Calculate the density of NH3 at 30° C and 5 atm pressure. (JEE 1978)

2. 4.215 g of metallic carbonate was heated in a hard glass tube, the CO2 evolved was found to measure
1336 mL at 27°C 700 mm of Hg pressure. What is the equivalent weight of the metal?
(JEE 1979)

3. 3.7g of a gas at 25°C occupied the same volume as 0.184 g of hydrogen at 17°C and at the same
pressure. What is the molecular weight of the gas? (JEE 1979)

4. A hydrocarbon contains 10.5g of carbon per gram of hydrogen. 1L of the vapour of the hydrocarbon at
127°C and 1 atm pressure weighs 2.8 g. Find the molecular formula of the hydrocarbon.
(JEE 1980)

5. The pressure in a bulb dropped from 2000 to 1500 mm of mercury in 47 min when the contained
oxygen leaked through a small hole. The bulb was then evacuated. A mixture of oxygen and another
gas of molecular weight 79 in the molar ratio of 1:1 at a total pressure of 4000 mm of mercury was
introduced. Find the molar ratio of the two gases remaining in the bulb after a period of 74 min.
(JEE 1981)

6. Calculate the average kinetic energy, in joule per molecule in 8.0g of methane at 27°C.
(JEE 1982)

7. At room temperature, ammonia gas at 1 atm pressure and hydrogen chloride gas at p atm pressure are
allowed to effuse through identical pin holes from opposite ends of a glass tube of one metre length and
of uniform cross-section. Ammonium chloride is first formed at a distance 60cm from the end through
which HCl gas is sent in. What is the value of p? (JEE 1982)

8. When 2g of a gas A is introduced into an evacuated flask kept at 25°C, the pressure is found to be one
atmosphere. If 3g of another gas B is then added to the same flask,the total pressure becomes 1.5 atm.
Assuming ideal gas behaviour, calculate the ratio of the molecular weight MA : MB.
(JEE 1983)

9. Oxygen is present in one litre flask at a pressure of 7.6 × 10-10 mm Hg. Calculate the number of Oxygen
molecules in the flask at 0°C. (JEE 1983)

10. Give reasons for the following in one or two sentences.


(i) A bottle of liquor ammonia should be cooled before opening the stopper.
(ii) Equal volumes of gases contain equal number of moles. (JEE
1983)

11. Calculate the root mean square velocity of Ozone kept in a closed vessel at 20°C and 82 cm mercury
pressure. (JEE 1985)

CENTERS: MUMBAI / DELHI / AKOLA / KOLKATA / LUCKNOW / NASHIK / GOA #


GASEOUS ST AT E Rg - 2015 - 2017
12. A spherical balloon of 21 cm diameter is to be filled up with hydrogen at NTP from a cylinder
containing the gas at 20 atmospheres at 27°C. If the cylinder can hold 2.82 L of water, calculate the
number of balloons that can be filled up. (JEE 1987)

13. The average velocity at T1 K and the most probable at T 2 K of CO2 gas is 9.0 ×104 cms-1. Calculate the
values of T1 and T2. (JEE 1990)

14. Calculate the volume occupied by 5.0 g of acetylene gas at 50°C and 740 mm pressure.
(JEE 1991)

15. At 27° C, hydrogen is leaked through a tiny hole into a vessel for 20 min. Another unknown gas at the
same temperature and pressure as that of hydrogen is leaked through same hole for 20min. After the
effusion of the gases the mixture exerts a pressure of 6 atm. The hydrogen content of the mixture is 0.7
mole. If the volume of the container is 3 L. What is the molecular weight of the unknown gas?
(JEE 1992)

16. At room temperature, the following reaction proceeds nearly to completion.


2NO + O2 → 2NO2 → N2O4
The dimer, N2O4, solidifies at 262 K. A 250mL flask and a 100mL flask are separated by a stopcock. At
300 K, the nitric oxide in the larger flask exerts a pressure of 1.053 atm and the smaller one contains
oxygen at 0.789 atm. The gases are mixed by opening the stopcock and after the end of the reaction the
flasks are cooled to 220 K. Neglecting the vapour pressure of the dimer, find out the pressure and
composition of the gas remaining at 220 K. (Assume the gases to behave ideally).
(JEE 1992)

17. A gas bulb of 1 L capacity contains 2.0 ×1021 molecules of nitrogen exerting a pressure of 7.57 × 10 3
Nm-2. Calculate the root mean square speed and the temperature of the gas molecules. If the ratio of the
most probable speed to root mean square speed is 0.82, calculate the most probable speed for these
molecules at this temperature. (JEE 1993)

18. A 4 : 1 molar mixture of He and CH4 is contained in a vessel at 20 bar pressure. Due to a hole in the
vessel, the gas mixture leaks out. What is the composition of the mixture effusing out initially?
(JEE 1994)

19. An LPG (liquefied petroleum gas) cylinder weighs 14.8 kg when empty. When full it weighs 29.0 kg
and shows a pressure of 2.5 atm. In the course of use at 27°C, the weight of the full cylinder reduces to
23.2 kg. Find out the volume of the gas in cubic meters used up at the normal usage conditions, and the
final pressure inside the cylinder. Assume LPG to the n-butane with normal boiling point of 0°C.
(JEE 1994)

20. A mixture of C2H6 and C2H4 occupies 40L at 1.00 atm and at 400K. The mixture reacts completely with
130g of O2 to produce CO2 and H2O. Assuming ideal gas behaviour, calculate the mole fractions of C2H4
and C2H6 in the mixture. (JEE 1995)

21. The composition of the equilibrium mixture (Cl2⇌ 2Cl) which is attained at 1200°C, is determined by
measuring the rate of effusion through a pin-hole. It is observed that at 1.80mm Hg pressure, the
mixture effuses 1.16 times as fast as krypton effuses under the same conditions. Calculate the fraction of
chlorine molecules dissociated into atoms (atomic weight of Kr = 84)
(JEE 1995)
22. A mixture of ideal gases is cooled up to liquid helium temperature (4.22 K) to form an ideal solution. Is
this statement true or false? Justify your answer in not more than two lines.
(JEE 1996)

CENTERS: MUMBAI / DELHI / AKOLA / KOLKATA / LUCKNOW / NASHIK / GOA #


GASEOUS ST AT E Rg - 2015 - 2017

23. An evacuated glass vessel weight 50.0 g when empty 148.0 g when filled with a liquid of density 0.98 g
ml-1 and 50.5 g when filled with an ideal gas at 760 mm Hg at 300 K. Determine the molar mass of the
gas. (JEE 1998)

24. Using vander Waals equation calculate the constant ‘a’ when two moles of a gas confined in a four litre
Flask exert a pressure of 11.0 atm at a temperature of 300 K. The value of ‘b’ is 0.05 L mol-1.
(JEE 1998)

25. (a) One mole of nitrogen gas at 0.8 atm takes 38 s to diffuse through a pin-hole. Whereas one mole of
an unknown compound of xenon with fluorine at 1.6 atm takes 57 s to diffuse through the same hole.
Calculate the molecular formula of the compound.( Given M (Xe) = 131 g/mole)
(b) The pressure exerted by 12g of an ideal gas at temperature t° C in a vessel of volume V litre is one
atm. When the temperature is increased by 10° C at the same volume, the pressure increases by
10%. Calculate the temperature ‘t’ and volume V. (Molecular weight of the gas = 120)
(JEE 1999)

26. Calculate the pressure exerted by one mole of CO2 gas at 273 K if the van der Waals’ constant a = 3.592
dm6 atm mol-2. Assume that the volume occupied by CO2 molecules is negligible.
(JEE 2000)

27. The compression factor (compressibility factor) for one mole of a van der Waals gas at 0° C and 100
atm pressure is found to be 0.5. Assuming that the volume of a gas molecule is negligible, calculate the
van der Waals’ constant ‘a’. (JEE 2001)

28. The density of the vapour of a substance at 1 atm pressure and 500K is 0.36 kg m -3. The vapour effuses
through a small hole at a rate of 1.33 times faster than oxygen under the same condition.
(a) Determine, (i) molecular weight (ii) molar volume (iii) compression factor (z) for the vapour
and (iv) which forces among the gas molecules are dominating, the attractive or the repulsive?
(b) If the vapour behaves ideally at 1000 K, determine the average translational kinetic energy of a
molecule. (JEE 2002)

29. The average velocity of gas molecules is 400 ms-1, find the rms velocity of the gas. (JEE 2003)

30. At 400 K, the root mean square (rms) speed of a gas A (molecular weight = 40) is equal to the most
probable speed of gas Y at 60 K. The molecular weight of the gas Y is ? (JEE 2009)

CENTERS: MUMBAI / DELHI / AKOLA / KOLKATA / LUCKNOW / NASHIK / GOA #


GASEOUS ST AT E Rg - 2015 - 2017

9. EXPERTISE ATTAINERS

1. A vessel contains 10 g of I2(s) and N2 at a pressure of 10 atm at 25°C. The volume of the vessel is one
litre. If this vessel is connected to a 40 litre empty vessel and the temperature of the first vessel is
increased to 250°C and of the second to 200°C. Calculate the final pressure in vessels.

2. A column of Hg of 10 cm in length is contained in the middle of a narrow 1 m long tube which is


closed at both ends. Both the halves of the tube contained air at a pressure 76 cm of Hg. By what
distance will the column of Hg be displaced if the tube is held vertical?

3. In a space shuttle the CO2 output per astronaut has been estimated as 44g/hr. An experimental
catalytic converter reduces CO2 at a rate of 600 ml STP per minute into H 2O. What fraction of time
would such a converter have to operate in order to keep up the CO2 output of one astronaut?

4. A compound exists in the gaseous state both as a monomer (A) and dimer (A2). The molecular weight
of the monomer is 48. In an experiment, 96 g of the compound was confined in a vessel of volume
33.6 litres and heated to 273°C. Calculate the pressure developed, if the compound exists as a dimer to
the extent of 50 per cent by weight, under these conditions (R = 0.082)

5. A mixture of H2O vapour, CO2 and N2 was trapped in a glass apparatus with a volume of 0.731 ml.
The pressure of total mixture was 1.74 mm of Hg at 23°C. The sample was transferred to a bulb in
contact with dry ice (–75°C) so that (H 2O)v are frozen out. When the sample returned to normal value
of temperature, pressure was 1.32 mm of Hg. The sample was then transferred to a bulb in contact
with liquid N2 (–95°C) to freeze out CO2. The measured, pressure was 0.53 mm of Hg at normal
temperature. How many moles of each constituent are in mixture?

6. A 50 litre vessel is equally divided into three parts with the help of two stationary semi permeable
membrane (SPM). The vessel contains 60 g H 2 gas in the left chamber, 160 g O 2 in the middle & 140
g N2 in the right one. The left SPM allows transfer of only H 2 gas while the right one allows the
transfer of both H2 & N2. Calculate the final ratio of pressure in the three chambers

7. A closed vertical cylinder is divided into two parts by a frictionless piston, each part contains 1 mole
of air . At 27 ºC the volume of the upper part is 4 times than that of the lower part. Calculate the
temperature when volume of the upper part will be three times than that of the lower part.

8. One mole of an ideal gas is subjected to a process in which P = 1 8.21 V where P is in atm & V in litre.
If the process is operating from 1 atm to finally 10 atm (no higher pressure achieved during the
process) then what would be the maximum temperature obtained & at what instant will it occur in the
process.

9. A barometer does not read correctly because of the presence of a small amount of air trapped
above the mercury column. At a pressure of 755 mm of Hg, the barometer reads 748 mm of Hg, and
at 740 mm of Hg, it reads 736 mm of Hg. What is the actual pressure if the barometer reads 750
mm of Hg at a constant temperature

10. A 100 feet long classroom maintains seating rows after every 10 feet and has doors on both front and
back sides. A N2O (laughing gas) gas cylinder and a methane (tear gas) cylinder were opened
simultaneously at the front and the back door, respectively.
(a) Student of which row, from front, will first get rid off tear ?

CENTERS: MUMBAI / DELHI / AKOLA / KOLKATA / LUCKNOW / NASHIK / GOA #


GASEOUS ST AT E Rg - 2015 - 2017
(b) Student of which row, from front, will first shed tear while laughing? Assume, both the gases
were present at the same temperature and pressure and have similar valve dimensions.

11. A one litre flask was evacuated, 5.0 g of liquid ammonia were added to the flask, and the flask was
sealed with a stop cork of radius 1.00 cm. When the ammonia gas in the flask warms up to 21� C,
will the cork be blown out of the mouth of the flask ? Assume that the cork is holding the mouth of
the flask with a force of 200N.

12. What pressure difference must be generated across the length of a 15 cm vertical drinking straw in
order to drink a water like liquid of density 1.0 g/cc?

13. A stockroom supervisor measured the content of a partially filled100 L acetone drum on a day when
the temperature was 18� C and the atmospheric pressure was 750 mm of Hg and found that 60 L
solvent remained. After tightly sealing the drum, an assistant dropped the drum while carrying it
upstairs to the organic laboratory. The drum was dented and its internal volume was decreased to 80
L. What is the total pressure inside the drum after the accident? The vapour pressure of acetone at
18� C is 400 mm of Hg.

14. A mixture of C5H8 and H2 gas was placed in a container at a constant temperature. Initial volume of
mixture was 10 ml. Sparking the mixture caused hydrogenation reaction
C5 H 8 ( g ) + 2 H 2 ( g ) � C5 H12 ( g )
Due to above reaction ( on cooling) volume decreases by 6 ml. Excess of O 2 was then added and on
sparking the volume further decreased by 15 ml. Calculate
(i) Volume of each gas in original mixture. (ii) Mole fraction of C5H8 in original mixture.

15. The respiration of the suspension of yeast cells was measured by observing the decrease in pressure of
gas above the cell suspension. The apparatus show that the gas was confined to a constant volume, 16
ml and the entire pressure change was caused by uptake of O 2 by the cells. The pressure was
measured in a manometer, the fluid of which had a density of 1.034 gm/ml. The entire apparatus was
immersed in a thermostat at 37°C. In a 30 min. observation period the fluid in open side of
manometer dropped 37 mm. neglecting the solubility of O2 in yeast suspension, compute rate of O2
consumed by the cells in lt of O2 (S.T.P.) per hour.

16. Find the critical constant (Pc, Vc and Tc) in terms of A and B, also find compressibility factor (z) for
A 2B
the following equation of state. PV = RT – V + 2
V
where A and B are constant, P = pressure and V = molar volume.

17. A thin tube of uniform cross section is sealed at both ends. It lies horizontally. The middle 5 cm
containing Hg and the two equal ends containing air at the same pressure P 0. When the tube is held at
an angle 60° with the vertical, the length of air column above and below the mercury are 46 and 44.5
cm respectively. Calculate the pressure P0 in cm of Hg (the temperature of the system is kept at
30°C).

18. Inside the spherical glass flask A of radius 1 m containing 300g of H 2 there was a rubber balloon B
containing some N2. Inside the balloon B was another rubber balloon C containing some O2. At 27°C
it was found that the balloon B had a radius of 60 cm and balloon C had a radius of 30 cm. Calculate
total weight of gases inside the flask. If another 50 gm of H 2 was not introduced in flask A, what
would be the volume of B & C. If the flask is now dipped in a bath containing acetone and dry ice (–
78°C) for sufficient time the temperature of the entire contents would reacts the bath temperature.

CENTERS: MUMBAI / DELHI / AKOLA / KOLKATA / LUCKNOW / NASHIK / GOA #


GASEOUS ST AT E Rg - 2015 - 2017
Assuming that at this temperature rubber cracks up and both the balloons burst what would be
pressure inside flask.

19. A mixture of CH4 & O2 is used as an optimal fuel if O 2 is present in thrice the amount required
theoretically for combustion of CH4. Calculate number of effusions steps required to convert a
mixture containing 1 part of CH4 in 193 parts mixture (parts by volume). If calorific value (heat
evolved when 1 mole is burnt) of CH 4 is 100 cal/mole & if after each effusion 90% of CH 4 is
collected, find out what initial mole of each gas in initial mixture required for producing 1000 cal of
energy after processing. [Given (0.9)5 = 0.6]

20. Find out the percentage of molecule in a gas at 1 atm and 27°C whose kinetic energy lies in the
3
range of E and E + 0.01E. [Where E = kT]
2

21. If volume occupied by CO2 molecules is negligible, then calculate the pressure exerted by one mole
of CO2 gas at 273 K. a = 3.592 atm litre2 mole−2.

CENTERS: MUMBAI / DELHI / AKOLA / KOLKATA / LUCKNOW / NASHIK / GOA #


GASEOUS ST AT E Rg - 2015 - 2017

10. Answer Key


Foundation Builders (objective)

CENTERS: MUMBAI / DELHI / AKOLA / KOLKATA / LUCKNOW / NASHIK / GOA #


GASEOUS ST AT E Rg - 2015 - 2017
1. C 17. A 33. C 49. B 65. B 81. B
2. A 18. D 34. A 50. D 66. B 82. A
3. C 19. B 35. A 51. A 67. C 83. B
4. A 20. C 36. A 52. B 68. D 84. A
5. A 21. D 37. A 53. A 69. C 85. A
6. B 22. B 38. A 54. A 70. C 86. B
7. C 23. B 39. C 55. C 71. B 87. C
8. C 24. A 40. D 56. D 72. B 88. C
9. D 25. C 41. C 57. A 73. C 89. B
10. A 26. C 42. C 58. B 74. B 90. C
11. D 27. A 43. A 59. D 75. A 91. C
12. A 28. C 44. A 60. D 76. C 92. C
13. A 29. A 45. C 61. B 77. A 93. C
14. D 30. B 46. A 62. C 78. D
15. D 31. A 47. D 63. D 79. D
16. B 32. C 48. B 64. B 80. A

Foundation Builders (Subjective)

1. Will burst 24 C2H4


2 a) 25.5 litre, b)1.5124 g/litre 25 C2H6O
3 a) 29.55g, b) 1051.4 mm of Hg 26 C3 H 8
4 750 K 27 Vol. of CO = 5 ml, Vol. of CH4 = 2 ml ,
5 68.7 % Vol. of N2 = 3 ml
6 100.67ml 28 0.638 , 0.262 , 0.1
7 9.08 cm
29 C2 H 4
8 4.93atm
30 a)900 cals b)51.29 cal
9 1 : 2 : 4, 2 : 3 : 4
31 236°C
10 89.58ml
32 1.83 x 10 3 m/sec
11 400.2K
33 URMS = 493 m/s ,Ump = 403m/s
12 66.7 atm
Uav=454.4 m/s
13 Ptotal = 27.54×105 N/m2 , Pfinal =
34 RMS = 2.77 m/s,
19.66×105N/m2
average speed = 2.716 m/s
14 94.54ml
35 942 m /s ; 71.2 K
15 po2 = 247mm, P= 427mm,
36 1.62Å
16 SO3 =0.6, SO2 =0.265 and O2 =0.135 atm 37 1.47  10–8 cm
17 2.19 atm 38 58.997 cm3
18 24.65 39 (a) 2.479 × 103 kPa, (b) 2225.55 kPa
19 43.58 40 15.40 kg
1
1 � 41 521 K
20 �
3
� � 42 (i) B, (ii) C, (iii) A
�24 �
43 (i) 31.1 atm, (ii) 31.4 atm
21 32 : 4 2 :1
44 Molar vol = 0.1353 L/mol, Z = 0.6957
22 M=2
45 370 cc
23 C2H4

CENTERS: MUMBAI / DELHI / AKOLA / KOLKATA / LUCKNOW / NASHIK / GOA #


GASEOUS ST AT E Rg - 2015 - 2017

Get Equipped for IIT-JEE (MAIN)

SINGLE OPTION CORRECT


1.d 2. c 3.b 4. a 5. a 6. d 7. d
8. d 9. b 10.b 11.b 12.d 13. b 14.b
15. d 16. b 17.c 18.a 19.d 20. d 21.d
22. c 23. c 24.a 25.a 26.b 27. c 28.b
29. b 30. b
Get Equipped for IIT-JEE (ADVANCED)

SINGLE OPTION CORRECT


1. C 2. A 3.D 4. B 5. A 6. A 7. B
8. A 9. A 10.C 11.B 12.A 13. A 14.B
15. D 16. A 17. B

MORE THAN ONE OPTION CORRECT

1.A,B,C 2.B,D 3.B 4.A,B,C 5.C,D 6.A,C 7.A,B,D


8.B,C 9.B,C,D 10. A,C,D 11.B,C 12.A,C,D 13. A, B 14.AB
15. AC 16.AB

COMPREHENSION TYPE

PASSAGE−1 1. A 2. C 3. A 4. A 5. C
PASSAGE−2 1.B 2. D 3. A
PASSAGE−3 1.C 2. B 3. B 4. D
PASSAGE−4 1.B 2. D 3. B
PASSAGE−5 1.A 2. B 3. D 4. B 5. C
PASSAGE−6 1. A 2. B
PASSAGE – 7 1.D 2. B 3.C
PASSAGE – 8 1.C 2. B 3. B
PASSAGE – 9 1.D 2.B 3.C 4.A

CENTERS: MUMBAI / DELHI / AKOLA / KOLKATA / LUCKNOW / NASHIK / GOA #


MATCHING TYPE
1. A – p,s ; B – r, C – p,q; D – p,s
2. A – r ; B – s, C – q; D–p
3. A – r ; B – p, C – s; D–q
4. A  p, q; B  p, s; C  p, q; D  p, q, r

Window to IIT- JEE (MAIN)

Objective Question I
1. (c) 2. (d) 3. (c) 4. (a) 5. (b) 6. (c) 7. (c)
8. (c) 9.(a) 10. (d) 11. (d)

Window to IIT- JEE (ADVANCED)

Objective Question I
2. (a) 2. (b) 3. (a) 4. (b) 5. (b) 6. (b) 7. (c)
9. (d) 9.(a) 10. (c) 11. (b) 12. (c) 13. (b) 14. (a)
15. (d) 16. (a) 17. (d) 18. (c) 19. (b) 20. (d) 21. (c)
22. (c) 23. (c) 24. (b) 25. (c) 26. (d) 27. (c) 28. (c)
29. (a) 30. (a) 31. (b)

Objective questions II
1. (a,b) 2. (b) 3. (a,c) 4. (a,c,d)

Assertion and Reason


1. (a) 2. (d)

Match the Columns


1. A → p,s; B → r; C → p,q; D→r

Fill in the Blanks


1. 900 2. R 3. inversely, time 4. 0.25 5. 1 : 16 6. less

True/False
1. F 2. F 3. F 4. F

Integer Type Questions


1. (7)

Subjective Type Questions


1. 3.42 g L-1 2. 12.15 3. 41.32 g 4. C7 H8
5. n(O2) : n(g) = 1:1.18 6. 6.21�10-21 J molecule 7. 2.20 atm 8. M A : M B = 1: 3
9. 11. 390.2 ms -1 12. 10
2.7 �1010 molecule
13. T1 = 1682.5K, T2 = 2142 K 14. 5.23 L 15. 1020 gm
16. P(NO) = 0.221 atm 17. 496 ms -1 , 407 ms -1 18. n(He): n(CH4) = 8: 1
3
19. V=2.46 m , P = 1.48 atm 20. 0.34, 0.66 21. 0.14
22. False 23. 123 g/mole 2
24. 6.46 atm L mol -2
25. (a) XeF6 (b) t = -173�C, V = 0.82 L
26. 0.99atm 27. 1.25 atm L2mol-2
28. (a) (i) 18gm mol (ii) 50 L mol-1 (iii) 1.22 (iv) repulsive force (b) 2.07 �10-20 J
29. 434 ms -1 30. 4

Expertise Attainers

1. 0.4254 atm 2.3 cm 3.0.622 4. 2 atm


5. 2.1  10 , 3.1  10 , 1.7  10
–8 –8 –8
6. 4 : 7 : 5 7. 421.9 K
8. 10,000 K 9. 758 mm of Hg
10. (a) 4th row (b) 3rd row
11. Yes, since the force exerted by gas on cork is 226 N which is more than 200 N
12. 1.5 kPa 13. 1100 mm of Hg 14. (i) 4 ml C5H8, 6 ml H2 (ii) 0.4
15. 064 × 10-4 ltr/hr
2 3 P V
16. VC = 6B ,TC = A , PC = A , compressibility factor = C C = 1
A 108B2 RTC 3
6RB
17. 75.4 cm 18. 0.768 atm

19. 10 Steps, 27.78 mol CH4, 5333.3 mol O2


20. 0.46% 21. 34.96 atm

Das könnte Ihnen auch gefallen